Property Quizzes

Ace your homework & exams now with Quizwiz!

Covenants recorded 40 years ago restricted all lots in a 90-lot subdivision to residential use only. Homes were constructed on all the lots. Four lots (lots A, B, C and D) at the edge of the subdivision adjoin the George W. Bush Parkway, at the time a two-lane road with residences and small businesses on both sides. Commercial development gradually intensified along the George W. Bush Parkway. Today the parkway adjoining the four lots and near the subdivision is a divided four-lane highway, lined with shopping centers, hotels, restaurants, automobile dealerships, and other businesses. Heavy automobile traffic and proximity to these businesses have made the four lots undesirable for residential purposes. If they retain their residential use only restriction, Lots A, B, C and D have a market value equal to approximately $60,000 but if the lots may be used for commercial purposes they have a market value equal to approximately $500,000. On four of the interior properties, homeowners conduct small-scale home businesses. The owners of lots A, B, C and D recently sold their lots to various real estate developers who are presently exploring commercial development possibilities. About one quarter of the homeowners in the subdivision support doing away with the residential purposes only covenant while the other three quarters oppose such action as they claim that the residential benefit is still alive and well. Some of the homeowners have brought an equitable declaratory judgment action against the new owners of lots A, B, C, and D, seeking a judgment that the restrictive covenant remains enforceable with an injunction to prevent any commercial development of lots A, B, C, and D. Will the court hold that the "residential only" covenant is still enforceable in equity? (A) Yes, the increased market value of the four lots for commercial purposes is not determinative of the outcome because the residential benefit can still be realized. (B) No, because the four lots are no longer suitable for residential use. (C) No, because no one has enforced the covenant against the four interior lot owners who conduct home businesses. (D) The outcome will depend upon whether horizontal privity was established when the covenants were created 40 years ago.

A is correct because the increased market value of the four lots for commercial purposes is not determinative of the outcome. The focus is on the impact within the subdivision and the benefit of residential purposes can still be achieved. Therefore, the residential only covenant is still enforceable in equity because the court would not find sufficient changed circumstances. B is incorrect because even if those four border lots don't realize the benefit, the subdivision as a whole still does. Those four lots don't legally justify enough for changed circumstances. C goes to waiver or abandonment, definitely not enough here for abandonment and as for waiver the character of a home business is significantly different from the character of commercial establishments and would still retain the residential benefits. D is incorrect because even if the covenant is enforced equitable relief is being sought and horizontal privity is not part of the elements to establish for equitable servitudes. That element would only matter if monetary damages were being sought since some courts say that changed circumstances cannot terminate a real covenant, only an equitable servitude.

On January 5, 2019, Chandler signed a written contract to purchase a house that Phoebe owns in a common law jurisdiction. The contract contained all the essential elements necessary to be binding, including the agreed upon purchase price of $197,000. The contract did not include any provisions addressing risk of loss. The contract provided that closing was to occur on or before April 19, 2019, and Chandler would take possession after closing. In March 2019, the house was destroyed by a fire started by some out-of-control college students on spring break. Chandler refused to complete the purchase of the house claiming the fire justified his refusal. Phoebe seeks specific performance of the contract. If Phoebe and Chandler have this dispute in a state that follows the common law, the resolution of the dispute would be: (A) Phoebe will prevail because of the doctrine of equitable conversion. Under this doctrine, Chandler is considered the owner of the property even though the closing has not yet occurred. Therefore, Phoebe has the right to specifically enforce the contract. (B) Chandler will prevail under the doctrine of equitable conversion. Under that doctrine, Phoebe is still considered the owner and therefore Chandler has the right to rescind the contract. (C) Chandler will prevail because Phoebe had the risk of loss for a total destruction of the improvements on the property. Therefore, Chandler has the right to rescind the contract. (D) Phoebe will prevail because Chandler should have had insurance for a total destruction of the improvements on the property. Therefore, Phoebe has the right to specifically enforce the contract.

A is the correct answer because at common law, the doctrine of equitable conversion informs the resolution of this issue. At the time the contract is signed, there is a split in legal and equitable title. The seller retains legal title while the buyer has equitable title. The buyer is viewed as the new owner of the property and bears the risk of loss. Phoebe is entitled to specific performance of the contract. B is incorrect because it misstates the doctrine of equitable conversion. C is incorrect because it does not correctly state the common law risk of loss. D is incorrect because whether or not a party has insurance on the property is irrelevant to the assessment of the risk of loss. In some common law jurisdictions the seller's insurance proceeds will be assigned to the buyer who must take the property in its damaged condition, but it is not part of the casualty loss analysis.

O conveys Blackacre to A by deed. A does not record. O then conveys Blackacre to B. B does not know about the O to A deed. B pays consideration. A records the deed. B records the deed. In an A v. B dispute, (A) A wins in a race and race-notice jurisdiction but B wins in a notice jurisdiction. (B) B wins in all jurisdictions because B has no notice of the deed to A. (C) A wins in all jurisdictions because A is the first to record. (D) B wins under the recording statute in all jurisdictions.

Answer choice A is correct. In a notice jurisdiction, at the time B takes the deed with no notice of A and pays good and valuable consideration, B wins priority without any need to record at all. A will prevail in the race and race-notice jurisdictions because A records first and B cannot fulfill the obligation in race and race-notice statutes to record first to take advantage of the recording statute priority exception. Bis incorrect because B cannot win in all jurisdictions because of the lack of recording before A. B's lack of notice is only part of the inquiry in a race-notice jurisdiction and B's lack of notice is completely irrelevant in a race jurisdiction. That is why answer D is incorrect. C is incorrect because A's first recording will only impact the race and race-notice jurisdictions. In the notice jurisdiction, recording after B takes the deed does not impact the outcome.

Penny lives in a house on a 20-acre parcel of land in a rural area. Her house is located on the rear portion of the land. She has a sewer line that runs from her house to the public sewer line and connects at the front part of the land. Penny sells the front 10 acres of her parcel to Sheldon. At the time she sells the front 10 acres to Sheldon, there is another public sewer line that has been installed along the rear of her retained parcel. While she could run a new sewer line and disconnect the old one, she prefers not to incur that cost. The deed of the front acreage to Sheldon says nothing of Penny's sewer line under the land. Penny wants to continue as she has been with the current line. Can she do so? (A) No, she did not expressly grant an easement in writing, therefore she is not legally entitled to continue using the sewer line. (B) Yes, although there is no easement in writing Penny obtained an implied easement by prior use because the continued use of the sewer line is reasonably necessary as is it more convenient and less expensive. (C) Yes, although there is no easement in writing Penny obtained an implied easement by necessity. (D) No, there is no easement in writing and Penny did not obtain an implied easement by prior use because the continued use of the sewer line is not strictly necessary.

B is correct because this is an attempted implied reservation of an easement by prior use and the majority view requires only reasonable necessity for the continued use and enjoyment of the now dominant estate. D is incorrect because only a small minority of states require an attempted implied reservation of an easement by prior use to meet a strict necessity requirement. The facts indicate that she could hook up to the city sewer line that runs along the rear of her retained land, but it is easier and cheaper to continue using the old line under the land that she sold. In the majority of states, that is the correct approach. A is incorrect because she would not have been granting an easement, she would have needed to expressly reserve one. C is incorrect because the implied easement by necessity only applies to landlocked parcels.

Michael owned Blackacre, a 100-acre tract of land. In a Special Warranty Deed, Michael conveyed Blackacre to Eleanor as a gift (Blackacre Gift Deed). Upon receipt of the Blackacre Gift Deed, Eleanor promptly put it in her desk drawer for safekeeping. As an annual gift, Michael continued to pay all of the property taxes on Blackacre. Neither Michael nor Eleanor ever recorded the Blackacre Gift Deed. Eleanor moved to Blackacre, where she built her dream home with her life savings. She subsequently died intestate. Upon learning of Eleanor's death, Michael sold Blackacre by a Special Warranty Deed to Chidi for cash (Blackacre Cash Deed). Chidi did not know either Michael or Eleanor prior to purchasing Blackacre from Michael. He promptly recorded the Blackacre Cash Deed. What impact, if any, would the recording have on the effect of the Blackacre Gift Deed? (A) The transfer of title is only valid if the deed is recorded. (B) The recording would give a rebuttable presumption of a valid execution, delivery, and acceptance. (C) Recording does not change the transfer of title, but it does provide a rebuttable presumption of a valid delivery. (D) It has no effect.

C is correct. The transfer of title from Michael to Eleanor was complete upon delivery of the deed and her acceptance of it. Her failure to record does not impact the transfer of title. The second conveyance to Chidi is completely void as Michael had nothing left to convey. Chidi's subsequent recording does not change the original transfer of title to Eleanor. The second recording does provide a rebuttable presumption of execution, delivery, and acceptance of the deed to Chidi. A is incorrect because the transfer of title occurs at the time of execution, delivery, and acceptance. Recording is not needed to transfer title. B is incorrect because although there is a presumption of a valid execution, delivery, and acceptance when a deed is recorded, that does not void the prior first in time deed which already transferred title to Eleanor. D is incorrect because the recording does have some impact in that it provides the rebuttable presumption of a valid delivery.

Lora owned the Mayfair Mall and leased one of the buildings to McBurgers Restaurants for a period of five years. Several months into the lease, McBurgers had trouble getting the floor drain to work properly, which resulted in water on the floor whenever employees put more than a gallon of water in the sink. The lease indicates that McBurgers took the premises "as is" and includes a provision imposing a duty on McBurgers "to make all necessary repairs to the premises." Which one of the following is true in most jurisdictions? (A) The implied warranty of habitability has probably been violated here, because this problem affects health and safety and may not be waived in the lease. (B) McBurgers will not be able to claim a violation of the implied warranty of habitability, because it did not suffer "constructive eviction." (C) McBurgers will not be able to claim a violation of the covenant of quiet enjoyment, because this problem has nothing to do with noise. (D) McBurgers would have a stronger claim for a violation of the quiet enjoyment covenant if the cause of the drainage problem is traced to a common area in Mayfair Mall outside the restaurant.

D is the correct answer. Remember that in most jurisdictions, the implied warranty of habitability applies only to residential dwellings, so both A and B answers can be discarded. B is also false because the IWH does not require a constructive eviction. The law generally gives commercial leases much more room to negotiate repair duties given the variety of situations presented and the relatively equal bargaining power. The covenant of quiet enjoyment does apply to commercial premises. However, answer C is incorrect because "quiet" enjoyment does not refer to noise only but rather to anything that disturbs the tenant's use and enjoyment of the premises. The interference must arise, however, from something the landlord causes or has a duty to remedy. Therefore, D is correct. In this case, Lora is not responsible for the condition of the leased premises under the terms of the lease. Nevertheless, landlords are responsible for the upkeep and repair of common areas, so if the problem stemmed from outside the leased premises, McBurgers should have a good case.

On December 16, 2018, Cook County conveyed a lot "to the City of Chicago for the life of George Lucas, remainder to the Lucas Museum of Narrative Art, but if said lot is not used for a permanent exhibition of the original Death Star, then the same shall revert to owner." What interest did the City acquire? (A) Life estate pur autre vie (B) Life estate defeasible (C) Fee simple determinable (D) Vested remainder in fee simple absolute

The correct answer is (A). The City acquired an interest in a life estate measured by the life of Lucas. (B) is incorrect because the life estate has no condition attached to it. (C) and (D) are incorrect because these answer choices fail to recognize that the interest at issue in this question is a present possessory interest in a life estate.

Lucky the Landlord and Tessa the Tenant enter a month-to-month lease on November 1, 2018 that specifies a rent of $1,200 per month. Tessa has always paid her rent on time and in full, but now she is considering moving home with her parents in order to save money. As of today's date (December 15th), the earliest date after which Tessa will no longer be responsible for rent would be: (A) December 31. (B) January 31. (C) January 1. (D) February 28.

The correct answer is (B). A periodic tenancy runs for a set period of time, in this case month-to-month. There are no facts present that suggest that this agreement created a year-to-year periodic tenancy. In a month-to-month periodic tenancy, either party can terminate the agreement upon notice given at least one month in advance. Here, Tessa decides to move on December 15. She would need to give at least one month's notice (at least 30 days) in order to avoid liability for rent. As a result, (A) and (C) must be incorrect, as these dates would only provide 16 or 17 days, respectively, of advance notice. (D) must also be incorrect, because there is no rule requiring that two months' notice (here, a period equal to 2x the length of the lease) be given in order to terminate a month-to-month periodic tenancy.

Last month, Arcadia City enacted an ordinance requiring all new and existing residential developments to include designated off-street parking for each dwelling unit. Daniela, a local real estate developer, thinks the requirement is excessive, cuts into her profit margins, and harms her business. She has decided to challenge the ordinance in court. What is Daniela's most persuasive argument in support of her claims? (A) The ordinance is a categorical or per se taking. (B) The ordinance is unconstitutional as applied to existing developments. (C) The ordinance is unconstitutional as applied to new developments. (D) The ordinance includes no financial incentives for developers to comply with its mandate.

The correct answer is (B). An ordinance that requires such substantial investment of existing parcels, without a grandfathering provision, is likely to be held unconstitutional. (A) is incorrect because the ordinance does not satisfy the categorical rules taken from the holdings of Loretto, Hadacheck, and Lucas. (C) is incorrect because the municipal police power vests the city with the power to zone for land use. Such legislation is subject to the rational basis standard of review, which examines whether the law is related to a legitimate state interest. Here, the state interest in street congestion justifies the ordinance. (D) is incorrect because an ordinance need not contain financial incentives in order to be applicable or enforceable against developers.

Kaya leases a loft apartment in downtown Arcadia City to law student Cliff for a term of three years at a monthly rent of $2,500. One year later, after he receives a clerkship out-of-state, Cliff "subleases, transfers, and assigns" the property to his classmate Thomas for "a period of one year from today's date." Thereafter, neither Thomas nor Cliff pays rent to Kaya. Does Kaya have any rights against Cliff? (A) Yes, Kaya has rights against Cliff because the transfer to Thomas is an assignment, and Thomas has not paid rent. (B) Yes, Kaya has rights against Cliff because the transfer to Thomas is a sublease, and Thomas has not paid rent. (C) No, Kaya does not have rights against Cliff because the transfer to Thomas is an assignment, and Thomas is liable to Kaya for the full rent. (D) No, Kaya does not have rights against Cliff because the transfer to Thomas is a sublease, and Thomas is liable to Kaya for the full rent.

The correct answer is (B). Cliff has a three-year lease and has transferred only one year to Thomas. Given that he has transferred less than his full interest—note that Cliff still has one year left on his lease after Thomas vacates—this transfer is characterized as a sublease. As the landlord and original tenant remain in both privity of estate and privity of contract, the tenant remains liable for rent. Therefore, (A) and (C) are incorrect. (D) is also incorrect because it confuses which party in a sublease arrangement is legally liable to the landlord for rent. The party on the hook for the rent in a sublease is always the original tenant (the sublessor), not the sublessee. See Ernst v. Conditt, 390 S.W.2d 703 (Tenn. 1964).

According to the writings of seventeenth-century German jurist Puffendorf, legal possession of wild animals can be satisfied only by: (A) Continuous Pursuit. (B) Physical possession / Seizure. (C) Minor injury to the animal (mild maiming). (D) Constructive possession.

The correct answer is (B). In Pierson v. Post, Justice Tompkins declined to follow Puffendorf's rule that legal possession ("occupancy") of ferae naturae can be satisfied only by "actual corporal possession." While bodily seizure alone can satisfy the rule of capture, other acts falling short of bodily seizure may also satisfy the rule, including "mortal wounding."

Siblings Landon and Mica inherited their mother's small real estate business in Arcadia City. The siblings' portfolio includes four single family homes and two apartments that are often rented as vacation rentals. In an attempt to get one of the houses rented, Landon posted on social media, stating: ''Single family home for rent to singles or childless couples.'' You are counsel in the Office of the Attorney General, and you notice the post while scrolling through your own feed. Does the post violate federal fair housing law? (A) Yes, because the Civil Rights Act of 1866 prohibits discriminatory advertising. (B) Yes, because the Fair Housing Act prohibits posts of this type. (C) No, because the Civil Rights Act is inapplicable to housing. (D) No, because of legislative exemptions to federal fair housing laws in the U.S.

The correct answer is (B). The social media posts are an advertisement, as defined by the Fair Housing Act, 42 U.S.C. 3601 et seq. The language ''to singles or childless couples'' expresses a clear preference to exclude families with children from the property, a form of discrimination based on familial status. (A) is incorrect because although the CRA prohibit discrimination, its protections do not extend to advertisements. (C) is incorrect because the Civil Rights Act applies to housing. (D) is incorrect because discriminatory ads are not subject to any of the FHA's exemptions (thus, the facts regarding the siblings' portfolio are irrelevant and do not trigger an exemption).

After their wedding in 2010, Juan Carlos and Jamie purchased a home, taking title as tenants by the entirety. Jamie now thinks the house no longer suits the family's needs, and she wants to move. Juan Carlos loves the house and has no interest in moving. Jamie sought advice from a friend, who suggested that Jamie pursue an action for partition in order to force the sale of the home. Will Jamie succeed? (A) Yes, because any interest holder can succeed on a petition for partition. (B) Yes, because Juan Carlos refuses to sell. (C) No, because a tenancy by the entirety cannot be severed. (D) No, because a tenancy by the entirety can be severed.

The correct answer is (C). A tenancy by the entirety, available only to married couples, cannot be unilaterally severed via an action for partition. Thus, (A) and (D) are incorrect. (B) is also incorrect, as Juan Carlos' refusal to sell has no bearing on the restriction on unilateral severance of property held by the entirety.

For two months, Tonya has been trying to trap a small animal who preys on her chicken coop and eats her flowers and plants. Earlier this week, she spotted the culprit—a coyote—and began chasing it. Tonya was able to capture the animal on her neighbor Patty's lawn. Patty is an avid animal rights advocate, and she is disturbed that Tonya may have harmed the animal. Patty decides to sue Tonya for the creature's return. Who is likely to prevail in this case? (A) Tonya, because the rule of capture confers ownership rights on the person who physically secures the wild animal. (B) Tonya, because the coyote was a nuisance who preyed on her chickens. (C) Patty, because once the coyote appeared on Patty's lawn, he became Patty's property. (D) Patty, because landowners do not have rights to animals on their property.

The correct answer is (C). The principle of ratione soliprovides that the owner of real property owns all wildlife found on her property, even if the animal in question is being chased. Both (A) and (B) are incorrect because Tonya has no rights to a wild animal that was neither captured, killed or mortally wounded by her on her own property. Once the animal crosses the property boundary, Tonya no longer has constructive possession and must yield to the constructive possession of her neighbors. (D) is incorrect because landowners have constructive possession over any wild animals on their property.

On December 16, 2018, Cook County conveyed a lot "to the City of Chicago for the life of George Lucas, remainder to the Lucas Museum of Narrative Art, but if said lot is not used for a permanent exhibition of the original Death Star, then the same shall revert to owner." What interest did the Museum acquire? (A) Fee simple determinable (B) Fee simple subject to a condition subsequent (C) Fee simple absolute (D) None of the above

The correct answer is (D). The Museum acquired a vested remainder subject to divestment, a future interest. (A), (B), and (C) are incorrect because each names a present possessory interest. The only present possessory interest in this grant is the life estate pur autre vie to the City of Chicago for the life of Lucas.

After weeks of sneaking past the "closed" signs and under the security gate at the nearby state nature preserve, Billie was finally able to successfully trap a rare wolverine that has been causing trouble with her chickens. Soon after she trapped the animal, she learned that it has been the subject of an extensive, yearlong search by scientists believing that the animal may be the first of its species to be seen in more than 200 years. The scientists demanded that Billie release the animal into their custody, and she refuses. Which party has the superior claim to the wolverine? (A) Billie has the superior claim because she trapped the wolverine. (B) Billie has the superior claim because the wolverine was a nuisance. (C) The scientists have the superior claim because of their investment in tracking the animal for a year. (D) The state has the superior claim because the wolverine was trapped on state land.

The correct answer is (D). The nature preserve is state property, and as per ratione soli, any wild animals located on this property are deemed to be in the constructive possession and ownership of the state. (A) is incorrect because, though Billie has trapped the wolverine, she has done so off her property. Moreover, Billie has engaged in trespass in her attempts to trap the animal, a fact that weakens her claim. (B) is incorrect because the animal's inconvenience to Billie does not grant her a superior claim over these other claimants. (C) is incorrect because, as per Pierson v. Post, substantial effort without capture is insufficient to establish legal possession.

Frank Goode executed a will that devised his farm Redacre "to Dr. Rodrigo Frausto so long as the property is used to provide medical care for farm animals, but if it ever ceases to be so used, to Adelphi County." True or False: At Goode's death, Adelphi County owns a vested remainder in fee simple determinable. (A) True, because of the durational language used in the grant. (B) True, because the property is still being used for medical care. (C) False, because the County's remainder is contingent upon the property being used for medical care for farm animals. (D) False, because the County's interest is not a remainder.

The correct answer is (D). This question requires students to distinguish between remainders and other possible future interests. A remainder is a future interest that may become active upon the natural expiration of the prior estate. The future interest here is not a remainder, because the County's interest can prematurely terminate Dr. Frausto's interest. (A), (B), and (C) are, therefore, incorrect. The future interest here is not a possibility of reverter because the possibility of reverter applies only to future interests retained by the transferor. The County holds a shifting executory interest in fee simple.

Olivia owned a tract of land bordered on the north by Green Road and on the south by Brown Road. There was a house on the north half of the property, which Olivia called Greenacre, with a driveway leading out to Green Road, and a dirt track, over the southern half of the property, called Brownacre, leading to Brown Road. In 2006, Olivia sold Greenacre to Andrew. Although the deed said nothing about an easement over Brownacre, Andrew frequently used the dirt track. After a mudslide closed Green Road in 2014, which remained closed for years, Andrew exclusively used the dirt track over Brownacre. Olivia then sold Brownacre to Betsy in 2016, who installed a fence across the dirt track to prevent Andrew from using it. In this jurisdiction, the statute of limitations for prescription is 10 years. Andrew sued to be able to continue to use the dirt track, and a court determined that Andrew had a legal easement over Brownacre. The county finally repaired and reopened Green Road, and Andrew stopped using the dirt track. Several months later, Betsy asked him if he ever intended to use the track again, and Andrew replied "No." Two years later, another mudslide closed Green Road, and Andrew wanted to use the easement again. Which of the following would be Betsy's best argument that the easement has terminated? (A) Abandonment (B) Estoppel (C) Release (D) Prescription

The correct answer is A. Abandonment requires a period of non-use accompanied by an intent to abandon. Here, Andrew stopped using it for a considerable period and indicated his intention to abandon it by saying he was giving it up. It was not a release because a release must be in writing and signed by Andrew to satisfy the Statute of Frauds, making answer C incorrect. Estoppel could terminate the easement if Betsy had changed her position in reliance in Andrew's promise to stop using it, but there is no indication that she did anything to her detriment. Thus, answer B is incorrect. Finally, an easement may be terminated by prescription, but that would require Betsy to block Andrew's use of the easement for the prescriptive period which we know is ten years. That didn't happen here. Therefore, answer D is incorrect.

Hope conveyed her property "to my partner Gloria for life, then to my daughter Frieda if then living." Which one of the following is true? (A) Frieda has a contingent remainder. (B) Frieda has a vested remainder. (C) Frieda has an executory interest. (D) Hope has a possibility of reverter.

The correct answer is A. Frieda's interest is a future interest because it follows Gloria's life estate. It is a remainder because it follows the natural termination of the preceding estate (i.e., it does not cut short Gloria's estate upon the happening of some event, which is why it is not an executory interest, making answer C incorrect). It is subject to a condition precedent; however, Frieda must be alive to take. Therefore, the remainder is contingent, not vested, which is why answer B is incorrect. Answer D is incorrect because Hope has a reversion, not a possibility of reverter. A possibility of reverter follows a fee simple determinable, which is not the estate at issue here.

Nick rented space in a shopping mall from Lorenzo at a monthly rental of $5,000 for a term of one year. Nick opened a jewelry store in the space, and his lease indicated that a jewelry store was the only use permitted. Clause 3 stated: "Tenant may not sublease or assign the leased premises without the permission of the Landlord, which may be denied for any reason." Lorenzo had Nick initial this provision separately. Three months into the lease, Nick realized that he could not make a go of it. He closed the store, but Lorenzo refused to terminate the lease. A couple of days later, Nick found a friend, Gina, who wanted to take over the lease and use the premises for a hair salon. Lorenzo refused stating that he thought this use might require too many renovations and damage to the premises. Nick then refused to pay any rent. The premises sat vacant for the next nine months. Lorenzo placed a small sign in the window, but no new potential tenants inquired about leasing the space. Lorenzo refused to take the keys back even though Nick offered them several times. Lorenzo then sued Nick for the entire amount of back rent. Which of the following is Nick's best argument? (A) Lorenzo breached his duty to mitigate. (B) Nick properly terminated the lease and owes at most one additional month's rent. (C) Lorenzo improperly refused to accept Gina as a subtenant. (D) Damages are limited to one additional month after termination of a lease.

The correct answer is A. In most jurisdictions, when a tenant breaches a lease, the landlord has a duty to mitigate damages. Mitigation requires making reasonable efforts to re-let the premises. In this case, Nick could argue that placing a small sign in the window was not sufficient. Lorenzo also refused a replacement tenant and did not advertise or use a broker to try to find a new tenant. Therefore, there is a good case for failure to mitigate. B is not correct because this is not a periodic tenancy. Nick cannot terminate a term-of-years lease before the end of the term. C is incorrect because the absolute clause in the lease would probably allow Nick to refuse to accept Gina. Even if Nick were required to be reasonable, Gina would change the permitted use and might cause damage, so Nick's refusal might be reasonable. C is also incorrect because Gina would be an assignee, not a subtenant. D is simply not a correct statement of law.

Pharmstore bought Lot 1 on the corner of 1st Avenue, a major thoroughfare in Anytown. It is currently zoned as R-3, which allows only residential housing (single-family, duplexes, or apartments). There is currently a small apartment building on the site, but Pharmstore wants to tear down this building and build a pharmacy there. There are other commercial uses up and down 1st Avenue and it has become a very busy street. Pharmstore has come to you to seek advice. Which of the following would you recommend? (A) Pharmstore should ask the planning and zoning commission to recommend a zoning amendment for the parcel. (B) Pharmstore should ask the board of zoning adjustment for a use variance for the parcel. (C) Pharmstore should ask the zoning administrator for a nonconforming use permit. (D) Pharmstore should ask the planning and zoning commission for a change to the comprehensive plan.

The correct answer is A. In this case, an amendment to change the zoning designation of this parcel from R-3 to some commercial use zone is the only viable option. In most jurisdictions, such an amendment would start with the planning and zoning commission, which would then recommend the change to the city council. A use variance, suggested in answer B, would not work because the test requires unreasonable hardship. Here, there is already an apartment building on the site, so presumably the land is being used productively (i.e., Pharmstore can collect rent from the tenants). As long as the property is able to be used productively, it's unlikely that Pharmstore could meet the hardship test. A nonconforming use permit, suggested by answer C, would work only if Pharmstore's commercial use pre-dated the zoning of the parcel as R-3, which is not the case. Answer D refers to the comprehensive plan. In most jurisdictions, the comprehensive plan is advisory only and would not have any actual effect on the permissible uses to be made of the parcel. Therefore, the best—and really only—way to achieve Pharmstore's objective is to seek a re-zoning of the parcel to commercial.

Poirot owned a large tract of ranch land, called Brushwood, which he bought in 2005 for $300,000 and turned into a game farm. Hunters pay large fees for the right to hunt his herd of captive elk on Brushwood. In 2016, however, the state passed a law prohibiting landowners from operating game farm operations due to the potential of such herds to spread diseases such as chronic wasting disease. Poirot estimates that the value of his ranch with its herd of elk just dropped from $3 million to about $500,000, which is its value for use as a vacation lodge. Each elk, which once brought in $5,000 as a hunting trophy, now must be sold for meat for about $800. Poirot decides to sue the state for a taking. In applying the Penn Central test, which of the following factors would weigh most strongly in favor of Poirot in this case? (A) Reciprocity of advantage (B) Reasonable return on investment-backed expectations (C) Character of the regulation (D) Conceptual severance, relevant parcel

The correct answer is A. The Penn Central test balances the impact on the property owner against the public interest, including the character of the regulation. Answer A is the best argument for Poirot; the fact that this regulation singles out game herd owners to bear the brunt of the health measure makes it seem unfair. This case is unlike zoning ordinances, for example, in which each landowner's burden is somewhat evened out by receiving the benefit of similar burdens on others. For a case that is somewhat similar to this fact pattern, see Kafka v. Montana Dep't of Fish, Wildlife & Parks, 201 P.3d 8 (Mont. 2008) (elk farm regulation did not constitute a taking). Answer B weighs in favor of the government here because the test focuses on whether the landowner can still make a reasonable return on his or her investment. Even though Poirot can't make as big of a profit as he expected, he can still get a reasonable return. The test focuses on what the landowner has left, rather than what was taken. Answer C also favors the government in this case because the regulation was enacted to prevent a noxious harm, the spread of chronic wasting disease. It is a harm-preventing, rather than benefit-conferring, type of regulation. Answer D has no application here; there is no dispute about what the relevant parcel is in this case.

A large amount of natural gas lies in a pool extending beneath both Blackacre, owned by Bart, and Whiteacre, owned by Wilma. Neither had ever tried to extract the gas, but recently the price increased to make extraction a valuable proposition. Bart decided to sink a well on Blackacre very near the property line to extract as much gas as he can. As the well extracted gas, some of the gas lying under Whiteacre migrated to Blackacre and was extracted as well. Wilma wants Bart to pay her for that amount of gas. If the jurisdiction applies the Rule of Capture to natural gas, which of the following is true? (A) Bart should not have to pay Wilma, because he was the first possessor of the gas. (B) Bart should have to pay Wilma, because she was the first possessor of the gas. (C) Bart should have to pay Wilma, because he is trespassing on her property. (D) Bart should have to pay Wilma, because under the ad coelum doctrine, she owns the resources beneath the surface of her land.

The correct answer is A. The Rule of Capture applies to unowned resources and gives ownership to the first possessor. Possession requires control. In the case of natural gas, only by extracting the gas can it be reduced to possession. Therefore, Bart was the first possessor and answer A is incorrect. C is not correct because trespass would require Bart to physically invade Wilma's property, which he is not doing. With regard to answer D, it is true that the ad coelum doctrine indicates that a landowner owns the resources beneath the surface, but with a "fugitive" resource like gas, that is only the case as long as it remains there. When it migrated off of Wilma's land, she lost any claim under the ad coelum doctrine.

Mutti owned a vintage Mercedes Benz automobile worth $200,000, which she kept in her garage, heavily insured. She wrote to her son, Seymour, a few months before his 21st birthday: "Seymour, I would like to give you the Mercedes for your birthday. I will keep it here for now because it requires safekeeping and I may want to drive it a few more times, but as soon as you have the capacity to take care of it properly, it will be yours." Mutti died with the Mercedes still in the garage. Her estate will be divided equally among her five children, but Seymour claims that the Mercedes should not be included in the estate because Mutti had already given it to him. Which of the following is true? (A) Mutti did not give Seymour a valid gift, due to lack of donative intent and delivery. (B) Mutti gave Seymour a valid gift inter vivos if the court accepts constructive delivery. (C) Mutti gave Seymour a valid gift causa mortis. (D) Mutti did not give Seymour a valid gift, due to lack of acceptance.

The correct answer is A. The gift will likely fail due to lack of donative intent (to make a present transfer of an interest) and delivery (at most, symbolic, which won't fly in this context). C cannot be correct because this was an attempt to give a gift inter vivos, rather than causa mortis, which is a gift in contemplation of death. Even though Mutti did in fact die, she did not give the gift because she was dying. A gift inter vivos requires donative intent, delivery, and acceptance. Acceptance is presumed for valuable gifts (although this may be rebutted). Here is there is no evidence of lack of acceptance, so D is not a good choice. There is also a big issue with donative intent. There must be an intent to make a present transfer of an interest in the property, even if possession is postponed. See, e.g., Gruen v. Gruen, 496 N.E.2d 869 (N.Y. 1986) where the gift of a painting was declared valid, even though possession was postponed until the donor died. In Gruen, however, the donor clearly specified he was making a gift presently (i.e., "I hereby gift you the painting") of a future interest in the painting versus the situation here, where there is merely an intent to make the gift in the future ("it will be yours"). Delivery is also suspect: there is no manual or constructive delivery. Manual would require the car to be actually sent to Seymour. Constructive would give Seymour control over the car (e.g., the keys and the title). The letter is only symbolic delivery at most, which many courts do not accept especially when other types (manual, constructive) are possible. So, if answer B said "if the courts accept symbolic delivery," it would be a possible answer, but there is no argument here for constructive delivery. This is also different from the Gruen case which allowed a symbolic delivery because the father retained a life estate (present possession) of the painting.

Georgia found some scrap metal in Harold's back yard. Thinking that Harold didn't want it, Georgia took the metal home and created a sculpture with it. When she showed the work at an exhibition, Harold recognized his metal and demanded the sculpture. Georgia offered to pay for the value of the metal, which was negligible. Harold claimed that he was entitled to the return of what was rightfully his property. Which of the following rules would be most relevant to this case? (A) The doctrine of accession (B) The rule of capture (C) The ad coelum doctrine (D) The law of finders

The correct answer is A. The law of accession gives property rights to one who significantly increases the value of another's property through labor. Georgia might be able to keep the property under this doctrine, especially if her acquisition of the scrap metal was innocent and the value of the scrap metal was negligible. B is incorrect because the rule of capture applies only to res nullius—unowned property—of a fugitive nature. C is incorrect because ad coelum refers to a landowner's rights to the airspace above the land and the resources below the surface, so it has no application to the personal property at issue here. And D is incorrect because the law of finders applies only to lost, mislaid, or abandoned property, which this is not. It might possibly be argued to be abandoned, but there is no indication that Harold signaled his intention to give up his ownership to the property and he remained in possession.

Which of the following is most likely to constitute a taking of property requiring just compensation under the federal constitution? (A) Because there is a hazardous waste site nearby, a federal agency issues an order allowing the cleanup contractor to install a small monitoring well on A's property which will help protect A's property as well as others. (B) The City of Hope bans building on hills with inclines greater than 15 percent grade because of the danger to people and houses below; as a result, B cannot build anywhere on her property. (C) State X prohibits C from disturbing Indian burial mounds on her 10-acre property; the mounds comprise 10 percent of the property's acreage. (D) D wants to subdivide his 10-acre tract into twenty residential lots; the City of Hope requires D to dedicate a street through the development for the public's use.

The correct answer is A. This is a physical occupation, which is one of the two "categorical" takings categories. In Loretto v. Teleprompter Manhattan CATV Corp., 458 U.S. 419 (1982), the Supreme Court held that any government action that authorizes a permanent physical occupation of property is a taking, without regard to the public interest involved. Therefore, the fact that it is "small" and will help protect A's property doesn't matter. Of course, the just compensation provided may be minimal, as in Loretto, but it is a taking. Answer B describes an ordinance that might fail the Lucas test because it seems to be a complete deprivation of economic value. However, it would probably fall under the nuisance exception to that test, allowing restrictions that do nothing more than duplicate "background principles" of nuisance and property law. The ordinance in answer C would be judged under the Penn Central balancing test. Because there is substantial economic value remaining and it does not require public access to the site, it would probably not constitute a taking. Finally, answer D is an example of an exaction which, under the test set out in Nollan and Dolan, requires that it have an "essential nexus" to the burden created by the new development and be "roughly proportional" to those burdens. In this case, a new development will create more traffic and the street requirement is reasonable. Therefore, answer A is the clearest case for a taking.

Betty Buyer bought a house in from Sally Seller, taking title by general warranty deed. The jurisdiction has no statutory disclosure requirements regarding the condition of the premises. Buyer inspected the property before closing but found nothing wrong. Shortly after moving in, however, she began to feel ill. After a few months, she discovered that there was mold growth behind the paneling in the living room. Seller had put up the paneling shortly before selling the property and admits she saw some mold at the time. Buyer received an estimate of $20,000 to fix the problem. Buyer now wants to recover that amount in damages from Seller. Which of the following is true? (A) Because the general rule is caveat emptor, Buyer cannot recover damages. (B) Buyer can recover damages, because the defect was not discoverable by reasonable inspection. (C) Buyer can recover damages, because property must be free of all material defects. (D) Buyer can recover damages based on seller's affirmative misrepresentation.

The correct answer is B. A seller must disclose material defects that are known to them and are not discoverable upon reasonable inspection. This is an exception to the rule of caveat emptor, making answer A incorrect. The mold was a material defect because it would have an impact of the value of the property. It was not discoverable because a reasonable inspection would not include tearing up paneling. It was known to seller because she admits that. Therefore, answer B is correct. Answer C is too broad; certainly property does not have to be free of material defects. There can be defects, as long as they are either patent or disclosed by seller, so that buyers can factor them into the offer price. Answer D is wrong because we have no evidence that seller made any representations regarding mold or the general quality of the house.

In the State of Euphoria, State Code Section 101: Limitation of Actions—Real Property, provides: "An action to recover title to or possession of real property shall be brought within eight years after the cause thereof accrued." The cause of action will only begin to accrue if there is good faith adverse possession. In 1999, Alpha owned 80 acres of farmland called Blackacre adjacent to 80 acres called Whiteacre, owned by Beta. A small stream meandered down the property line dividing the two pieces of land. In 2003, torrential rains and flooding caused the stream to swell dramatically and change course, which put about four acres of Blackacre (Parcel A) on Beta's side of the creek. Thereafter, Alpha did not bother to cross the creek to plant those four acres on Parcel A and they began to get overgrown with weeds. In 2006, Beta began planting corn on Parcel A. Alpha didn't object, so Beta continued to plant there every year after that. This required about six months of intermittent occupation of the parcel, to plant, fertilize, weed, and harvest the crop. In 2016, a new highway by-pass was built near the two properties, and Alpha decided to sell Blackacre to Gamma, who wanted to develop the land for industrial use. Gamma has raised a question about whether Alpha still owns Parcel A. Which element of adverse possession would be most likely to favor Alpha? (A) Beta's possession was not open and notorious. (B) Beta did not have the state of mind required in this jurisdiction. (C) Beta's possession was not continuous for the statutory period. (D) Beta's use of the parcel was not sufficient actual possession because there was no improvement by Beta.

The correct answer is B. Alpha's best bet is if this jurisdiction requires "good faith" as a state of mind, which a sizable minority of jurisdictions do. Beta seemed to know that the land was not his—he waited a few years after the flood before starting to farm it and did it only so long as Alpha didn't object. In good faith jurisdictions, the possessor must reasonably believe that the land is hers in order to qualify as an adverse possessor. We don't know if this jurisdiction requires good faith, but it is Alpha's best chance to defeat the adverse possession claim here. Answer A is incorrect because Beta's possession was clearly open and notorious—he did not try to hide his use of the site and it was fully visible to all. Answer C is incorrect because Beta did continually possess the parcel for the statutory period of eight years; even though Beta did not physically occupy the land for six months each year, the possession has to fit the nature of the property. Beta used the land as farmland is generally used, which is sufficient to put the true owner on notice of the claim to ownership. Similarly, D is incorrect because cultivation is one of the main ways property can be possessed; it certainly makes beneficial use of the property, which should be sufficient to claim adverse possession.

Public utility companies must acquire many easements over private land in order to get electricity from power plants or wind farms to the end users. In State A, public utilities are granted the right to use eminent domain to force transmission lines through private property if the landowner does not voluntarily sell the easement to the company. The statute provides for an independent appraiser to set the price, but if the landowner disputes the amount, a jury trial may be had on the issue of damages. Recently, utility companies have argued that this procedure is too slow and burdensome, and that they often end up overpaying for the easements. This year, the legislature enacted a new statute which allows utilities to pay a flat fee of $1 per linear foot. This is at the low end of recent valuations which have ranged as high as $5 per linear foot, depending on the type of land involved and the interference posed by the easement. However, the utilities argued that those high-end cases were aberrational and represented misinformed jury decisions. A private landowner group has brought suit, challenging the new statute as unconstitutional. Which of the following is the most likely outcome of this lawsuit? (A) The landowner group will win, because the statute fails the public use requirement. (B) The landowner group will win, because the statute fails the just compensation requirement. (C) The statute will be upheld, because the Takings Clause does not apply to these transactions. (D) The statute will be upheld, because this statute does not violate the constitutional Takings Clause provisions.

The correct answer is B. Answer C is wrong because this is a taking of a property interest in the form of an easement. A physical occupation of property such as this is a clear-cut taking. So the question is whether the Takings Clause is violated by this statute. Answer A is incorrect; the statute does not violate the public use requirement because a transfer of property from one private party to another is permissible as long as there is a public purpose. In this case, getting electricity to the public is a sufficient public purpose. The just compensation requirement, however, is not fulfilled by this statute. A property owner is entitled to the fair market value of the property taken, and by setting a flat rate of compensation for every taking, this statute does not guarantee that. Short-changing the owner to promote efficiency is not allowed. Therefore, answer B is correct and answer D is not.

Amie bought Unit 101 of Riverview Condominiums from Betty, who had lived there for ten years. Riverview Condominiums has a declaration of covenants which was recorded when Betty bought her unit. The covenants provide that they may be amended or supplemented by a majority vote of the Board of the Condominium Association. When Amie moved in, there were no restrictions in the recorded covenants on what could be stored in the carport assigned to each unit. Amie liked to race motorcycles, and she stored her motorcycle trailer in the carport. Recently, the Condominium Association Board voted to enact a new covenant, in accordance with the procedures for amendment. The new restriction allows only automobiles to be stored in the carports and specifically prohibits "campers, recreational vehicles, or trailers of any kind." Amie believes this is unfair because her trailer is not causing any harm. Which of the following would be your best advice about challenging this restriction? (A) Courts usually hold that restrictions such as this may not be challenged unless they are unconstitutional. (B) If the court requires reasonableness, Amie will probably have to show that the covenant is unreasonable in general rather than as applied to her. (C) Typically courts do not impose a standard of reasonableness on amendments to the covenants although they do require reasonableness for the original set of covenants. (D) The Condominium Association may not amend the covenants once a unit owner has moved into her condo without unanimous consent of all owners.

The correct answer is B. Court decisions are split on the issue of the standard of review of covenants and amendments to those covenants. Although some courts take a very deferential view of covenants on the basis that they are freely entered into by the parties, answer A is too broad. Even courts who use a deferential form of review will strike down covenants that violate public policy or are arbitrary and capricious in addition to those that violate the constitution. Many courts use a "reasonableness" standard of review, especially for amendments to covenants. Therefore, answer C is wrong because the standard is typically stricter for amendments than for restrictions in the original declaration. If the covenant is in the declaration, the owner knowingly agreed to the restriction by buying the property. Later amendments, however, can upset settled expectations. Answer D is also incorrect; courts allow for a less-than-unanimous amendment process. Therefore, the best answer is B. Courts that use a reasonableness test usually require a showing that the covenant is unreasonable in general rather than as applied to a particular case or individual. See Nahrstedt v. Lakeside Village Condom. Assoc., Inc., 878 P.2d 1275 (Cal. 1994).

Gramps wanted to give Blackacre to his two grandchildren, Nick and Nora. On Monday, he gave Nick a deed to Blackacre which stated "Gramps conveys to Nick a half-interest in Blackacre." On Tuesday, he gave a deed to Nora, which stated "Gramps conveys a half-interest in Blackacre to Nora, in joint tenancy with Nick." Nick died shortly thereafter, leaving a will giving all of his property to his wife, Wanda. Which of the following is true? (A) Wanda takes nothing, because of Nora's right of survivorship. (B) Wanda takes a half-interest in Blackacre as tenant in common with Nora. (C) Wanda takes nothing, because Gramps cannot convey Blackacre in this manner. (D) Wanda takes nothing, because Gramps intended only his grandchildren to benefit.

The correct answer is B. Gramps may have intended to create a joint tenancy with Nick and Nora, but he failed. He did not convey to Nick and Nora at the same time with the same instrument, thereby failing two of the required unities of time and title. In addition, his conveyance to Nick does not even mention a joint tenancy. So when he conveyed a half-interest to Nick, the result was a tenancy in common between Gramps and Nick. He then conveyed his remaining half-interest to Nora, creating a tenancy in common between Nick and Nora despite the language of the conveyance. When Nick died, then, his interest would go to Wanda under his will, resulting in a tenancy in common between Nora and Wanda. Answer A is therefore incorrect; there is no right of survivorship in a tenancy in common. C is incorrect because Gramps did succeed in conveying his interests, just not in joint tenancy. D can't be right; even if Grandpa intended only his grandchildren to benefit, he did not limit his conveyance to a life estate or in any other way. Therefore, they each took a half-interest in fee simple absolute, which is devisable, inheritable, and alienable. Certainly, at common law he should have said, "to Nick and his heirs,"but that language is no longer required and we assume that Nick was intended to have his half-interest in fee simple absolute.

In 2010, Whistler borrowed $50,000 from his mother, Ma, who lived nearby. He signed both a note and a mortgage on Blackacre, a rental property he owned, to secure the loan. Ma, however, did not record the mortgage at that time. On March 1, 2016, Whistler entered into a contract to sell Blackacre to Bono. Bono conducted a title search which of course did not reveal the mortgage to Ma and he was not told of her interest. On April 1, 2016, the sale closed. Bono paid full value for the property ($200,000) in exchange for a general warranty deed. Whistler told Ma he would pay her out of the sale proceeds, but he did not. Instead, he took the money and left for Europe. On April 15, 2016, Ma recorded her mortgage. On April 30, 2016, Bono recorded the general warranty deed. Ma now seeks to foreclose on Blackacre to satisfy her mortgage. Which of the following is true? (A) If this state has a race-notice recording act, Ma will not be able to foreclose because her interest was not recorded when Bono bought Blackacre. (B) If the state has a notice-type recording act, Ma will not be able to foreclose on Blackacre, even though she recorded first. (C) Unless the state has a pure race recording act, Ma will not be able to foreclose on Blackacre. (D) Regardless of the type of recording act, Ma's interest would not survive the transfer of title to Bono; her only remedy is to sue Whistler on the note.

The correct answer is B. In a recording act problem, draw a little diagram showing when the conveyances and recordings occurred. In this case, Whistler conveyed to Ma, then Whistler conveyed to Bono, then Ma recorded, then Bono recorded. Because the conveyance to Ma was first in time, Ma will be able to foreclose unless Bono satisfies the requirements of the jurisdiction's recording act. If it is a notice act, Bono wins because he took without notice of the previous transaction. If it is a race-notice act, Bono loses because even though he took without notice, he must also record first, which he did not. Similarly, under a pure race statute, Bono loses because Ma recorded first. Therefore, answer A is incorrect because Ma would be able to foreclose in a race-notice state. Answers C and D are also incorrect for the same reason. Answer B is the only correct statement of the law.

In 2012, Kevin bought Lot 1 of Greenacres subdivision in the city of Anytown. When the subdivision was originally platted in 1927, the developer placed a covenant on all of the lots, restricting them to residential use only. In 1955, Anytown zoned the property R-1, which allows only single-family residential use. The street that Kevin's property is on, once a two-lane road, has now become a major thoroughfare, and several businesses and a large church now exist across the street from the subdivision. One resident on an inside street uses her house for a small day-care, and another resident is an accountant who receives clients at her house. Kevin would like to use his house on Lot 1 for a law office which he expects will have little traffic and no noise. He can prove that the house is worth $200,000 as a residence but about $300,000 if commercial use such as this is allowed. His neighbors in the Greenacres subdivision object, however, and say they intend to enforce the covenant against his proposed use. With regard to the zoning ordinance, which of the following is Kevin's best option? (A) Apply for a variance from the zoning ordinance. (B) Check to see if the ordinance allows for home offices by special permit or conditional use. (C) Seek to strike down the ordinance as a taking of property without just compensation. (D) Seek to have the home office allowed as a "nonconforming use."

The correct answer is B. Kevin's best option would be to see if the city allows special permits or conditional uses in this zone, which may be granted or allowed if the city is satisfied that conflicts with neighboring properties are minimized. If not, the only additional option would be for Kevin to seek re-zoning of the property. Kevin is unlikely to get a variance, as Answer A suggests; the test for a use variance typically requires unreasonable hardship. This property can still be used as a residence so it does not rise to the level of hardship usually required for variances. Answer C is incorrect because the level of deprivation here does not rise to the level required for a taking; in addition, the restriction was in place when Kevin bought the property so it did not upset his settled expectations. Answer D concerns the "nonconforming use" which occurs when a use pre-dates a zoning change, such that previous uses are grandfathered in. That does to describe this situation.

Which of the following is not a potential violation of the federal Fair Housing Act or the Civil Rights Act of 1866? (A) The landlord of a large apartment complex refuses to rent an apartment to A because she has two young children who might disturb the other residents. (B) The landlord of a large apartment complex refuses to rent an apartment to B because she is unmarried and lives with her boyfriend. (C) Green advertises in local newspaper: "For rent, furnished apartment in private white home." (D) The owner of a duplex refuses to allow her tenant, X, who is confined to a wheelchair, to build a ramp leading to the porch, even though X promises to restore the premises to their original condition after the lease terminates.

The correct answer is B. Neither the FHA nor the CRA applies class. The CRA protects only against racial discrimination, while the FHA protects against discrimination based on race, color, religion, national origin, sex, disability, and familial status. Answer A is incorrect because familial status is defined to mean persons with children and this is exactly what it is intended to prevent. The Fair Housing Act prohibits advertisements indicating a racial preference, either explicitly or by implication. Therefore, Mrs. Green's advertisement in Answer C violated the FHA. Answer D is also incorrect because the prohibition against disability discrimination requires a landlord to allow reasonable modifications of existing premises occupied by such person if modifications may be necessary to afford such person full enjoyment of the premises

Kim and Kanye were married and owned a vacation home called Greenacre as tenants by the entirety. One night, Kim had too much to drink and got into an accident. She hit a car driven by Talia, injuring her badly. Talia sued and obtained a $2,000,000 judgment against Kim. Because Kim had insufficient insurance to pay this amount, Talia wants to satisfy the judgment against Greenacre. Which of the following is true in most American jurisdictions that allow tenancy by the entirety? (A) Talia should be able to reach Kim's half-interest in Greenacre, because tort judgments are generally an exception to the rule against unilateral severance. (B) Talia will not be able to attach Greenacre at all, because property held in tenancy by the entirety is not subject to debts of one spouse only. (C) Talia will not be able to reach Greenacre now but can create a lien on the property, which she can enforce upon sale of the property. (D) Talia should be able to reach the entirety of Greenacre, because Kim was entitled to possession of the whole.

The correct answer is B. Property held in tenancy by the entirety cannot be reached by the creditors of one spouse only. See Sawada v. Endo or RBS Citizens v. Ouhrabka. Most jurisdictions do not make an exception for tort judgments, so A is incorrect. Answer C is not the rule in most jurisdictions that use TBE, with the exception of federal tax liens. Kim certainly was entitled to possession of the whole, as Answer D states, but the whole idea of tenancy by the entirety is to prevent one spouse from unilaterally encumbering the asset, so the right to possession is not relevant.

O conveyed property "to my son Adam for life, then to Adam's children." At the time of conveyance, Adam is alive and has two children, Kate and Julia, in high school. Which one of the following is true? (A) Kate and Julia have an executory interest. (B) Kate and Julia have a vested remainder, subject to open. (C) Kate and Julia have a contingent remainder. (D) The future interest in Kate and Julia is void under the Rule Against Perpetuities.

The correct answer is B. The children's interest follows the natural termination of the preceding life estate, so it is a remainder. It is not triggered by a stated event and is therefore not an executory interest, making answer Aincorrect. It is a class gift and all it takes to join the class (and thereby acquire a vested interest) is to be a child of Adam. So Kate and Julia have a vested remainder, but it is subject to open because more children could be born and join the class. (Some call this "subject to partial divestment.") Since the remainder is vested, answer option Cis incorrect. Answer D is incorrect because the remainder is valid under the Rule: you ask "when will we know who all the takers are?" When Adam dies, the class will close. So, because Adam is a life in being at the creation of the interest, the future interest is good under the Rule.

Alpha owns Whiteacre, adjacent to Blackacre, owned by Beta. A stream running through Blackacre has begun flooding frequently, and Beta has discovered that the problem is caused by a buildup of brush and mud acting as a dam downstream as the stream runs through Whiteacre. Alpha has refused to take any action and has refused to allow Beta to come on his land to clear the debris. The evidence indicates that correcting the problem would increase the value of both properties. One day, Beta got frustrated and sneaked onto Whiteacre with a chain saw and pickup and cleared the debris. Alpha sued Beta for trespass and sought punitive damages. Assume that the court holds for Beta, finding no actionable trespass in this case. Which of the following property policies would be furthered most by this holding? (A) Certainty (B) Economic efficiency (C) Peace and order (D) Personhood

The correct answer is B. The facts state that both properties were made more valuable by Beta's actions. Economic efficiency favors a rule that makes property more productive or more valuable overall. Certainty would not be furthered by this rule, however, making answer A incorrect. The most certain rule is a bright line that allows the owner to exclude for any reason and would not have the rule vary based on the reasons for the intrusion. Peace and order would also not be served by this outcome because trespass carries with it the risk that the landowner will try to eject the trespasser by force. C is therefore incorrect. Personhood would also not favor this outcome making D incorrect because Alpha's privacy and security would be threatened by this trespass.

On July 1, tenant Tom leased an apartment from landlord Lu on a month-to-month basis for $1,000/month for a minimum period of ten months. The two parties agreed with a handshake and a verbal agreement rather than writing anything down. When Tom moved in, the apartment was in good shape. On November 1, however, Tom's toilet stopped working and the kitchen sink clogged up. Tom called Lu immediately and Lu said he would "take care of it." Three days later, however, nothing had been done, so Tom notified Lu in writing that he wanted action quickly on these defects. In response, Lu came by and messed around with the toilet and sink but failed to fix them. He said he would need to "call my friend Brian. He can fix this stuff." Tom could not cook because he couldn't use the sink and had to use his neighbor's toilet or go down to the lobby if his neighbor wasn't home. By December 1, Lu still had not addressed the problems, so Tom didn't pay his rent. January arrived and the problems were still not fixed, so Tom did not pay January rent either. Lu finally fixed the problem on January 15 and now wants Tom to pay the back rent. Which of the following doctrines is Tom's best defense to this claim? (A) Covenant of quiet enjoyment (B) Implied warranty of habitability (C) Repair and deduct remedy (D) Statute of Frauds

The correct answer is B. The implied warranty of habitability requires the landlord to maintain the premises in a fit and habitable condition. A violation occurs if there is a material interference with habitable conditions, which this seems to be. In addition, the tenant must notify the landlord of the defect and allow a reasonable time to remedy it, which Tom did here. Therefore the IWH would be a viable defense. Answer A is incorrect because the covenant of quiet enjoyment does not apply. Tom did not sue the landlord to recover the difference between the amount due under the lease less the value of the leasehold with the defective condition, known as "difference money." Here, he simply withheld rent and that does not fit within this common law concept. Tom might have used the repair and deduct remedy for these violations, but it would require him to hire someone to fix the defects himself and then deduct the repair costs, which he did not do. Thus, Answer C is incorrect. The Statute of Frauds requires only leases in excess of a year to be in writing to be enforceable. Therefore, the Statute of Frauds won't help Tom here. (You might also note that exceptions like part performance might make the lease enforceable even if the Statute of Frauds did apply.) Answer D is incorrect.

Bianca conveyed Blackacre "to Carmen for life, then to Carmen's daughter Darcy if she reaches 21." Carmen is alive and Darcy is 15. Which of the following is true in most modern jurisdictions? (A) Darcy has a vested remainder, subject to divestment. (B) Darcy has a contingent remainder. (C) Darcy has an executory interest. (D) Darcy has a vested remainder.

The correct answer is B. The interest is a remainder because it follows the natural termination of the preceding life estate. It is contingent because it is subject to a condition precedent; Darcy takes only if she reaches 21. When she reaches 21, the interest will vest, but she is only 15. Therefore, answers A and D are incorrect. An executory interest cuts short or divests the previous estate upon the happening of some event. That will not happen here. The life estate will end only at Carmen's death, and C is incorrect because the executory interest cannot follow a life estate.

In the midst of a massive housecleaning project, Martha put an old chest of drawers near the curb so that the waste removal company could pick it up. George passed by in his pickup truck and saw the chest, which he thought he could refinish and make it look like new. After he got the chest home and started refinishing it, he found out it was a piece of Chippendale furniture, worth about $10,000. He displayed the piece on an antiques TV show and Martha saw that the chest was hers. Martha now sues George for replevin. What result is most likely? (A) Even though George was the first finder, Martha will win if she can prove that she is the true owner of the chest. (B) George will win, because the chest was abandoned by Martha. (C) Martha will win but will receive only the value of the chest in its original condition. (D) George will win, because the chest was mislaid or treasure trove.

The correct answer is B. The property in this case was not lost or mislaid but rather abandoned because Martha exhibited an intent to relinquish her possession over the chest by putting it out for trash pickup. In the case of lost or mislaid property, the true owner always retains the superior right of ownership and the finder's ownership is therefore conditional on the true owner not reclaiming it. With abandoned property, however, the true owner has relinquished her claim to the property so that the first possessor's ownership is no longer relative but absolute. Thus, answers A and C are incorrect. Answer D is incorrect because the property is clearly not treasure trove, which involves buried treasure, such as coins or jewels.

Ringo bought Blackacre, a one-acre lot with a large house and a big shed in back. The area is zoned single-family residential. Ringo would like to convert the shed into a recording studio. He would have a few artists record there every month. The shed would be soundproofed, so that the neighbors would not be bothered by noise. The residential zone, however, does not allow any kind of commercial activity, so Ringo applied to the Board of Zoning Adjustment for a variance. Which of the following is most likely true? (A) A use variance such as this is typically easier to get than an area variance. (B) Ringo's request does not meet the variance test of unnecessary hardship. (C) If Ringo receives a variance, Ringo's neighbor will not be able to sue for a nuisance. (D) To determine whether to grant the variance, the Board will balance the utility of Ringo's use against the harm caused by the use.

The correct answer is B. The test for a variance in most jurisdictions requires "unnecessary hardship" which examines the burden on the landowner if the variance is not granted versus the burden on the neighborhood if the variance is granted. Hardship alone is not enough; some hardship may be necessary to further the objectives of the zoning ordinance. Answer A is incorrect because typically courts use an easier test (such as "practical difficulties") for an area variance. Answer C is incorrect because zoning permission does not preclude a nuisance case. It may help Ringo establish the reasonableness of his use, but it will not decide the case. Answer D states the test incorrectly; the "utility" or value of Ringo's use is not at issue. Rather the question is what harm he will suffer if he is unable to get the variance. Usually courts require that the harm be relatively high; in some jurisdictions, Ringo would have to show he is unable to make productive use of the property without the variance. Therefore, most likely Ringo does not meet the variance test, as answer B states.

Harry and Wilma married in 2005 and lived happily for years in a house on Blackacre. Harry owned Blackacre before he met Wilma and, after they married, the property remained titled in Harry's name alone. In 2013, the couple had a big fight and Harry moved out. After the couple had been separated for a year, Harry decided to sell Blackacre. Wilma adamantly objected, pointing out that it was still her home. Harry noted that the title was still in his name only, so he sold the property to Xavier. Which of the following is false? (A) If Blackacre were homestead property, Harry could not sell clear title to it without Wilma's consent. (B) If this state has an elective share statute, Wilma could prevent the sale to protect her elective share rights. (C) If this is a community property state, Blackacre may have become community property, because it was used in the marriage by the couple. (D) If dower rights exist in this state, Harry could not sell clear title to Blackacre without Wilma's consent.

The correct answer is B. This question asks you to compare various forms of spousal protection. In a separate property state, property titled in the name of one spouse is not typically subject to the rights or interests of the other spouse until divorce or death. So, there are limited ways for Wilma to prevent the sale in a separate property state. Answer A is true. If the property is classified as homestead, it can't be sold without Wilma's consent, and that makes Aincorrect. Answer B, however, is false (and therefore the correct answer to the question). Elective share provisions give surviving spouses the right to take a portion of the deceased spouse's estate, regardless of the will. However, that right usually applies only to property owned by the decedent spouse at death. Prior to death, it does not give Wilma any right to Harry's property. Answer C requires you consider the difference if this were a community property state. Blackacre would be considered separate property at first because Harry owned it prior to the marriage. However, separate property may lose its character if it is used by the couple during the marriage. So, Answer C is true and incorrect. Answer D is also true; Wilma's dower right attaches to any property owned by Harry during the marriage. It is therefore very powerful marital protection because Harry can't destroy Wilma's dower interest by selling Blackacre even though it's titled in his name only unless Wilma waives her interest. D is incorrect.

Jacqui had a heart attack and was in intensive care in the hospital. Her niece Nala visited her. Jacqui told Nala: "Look, I think that I am going to die. Here is the key to my safe deposit box at the bank. I want you to have all of my jewelry, which I keep in that box. I know you will take care of it." Nala took the key and, later that day, retrieved the jewelry from the box. She brought it to the hospital, and Jacqui manually handed over the items from the box. After some surgery and a few weeks in the hospital, Jacqui was released. The doctors believe she will be fine if she eats right and exercises properly. Shortly thereafter, Jacqui asked Nala to return her jewelry; Nala wants to keep it. Which of the following is true? (A) Jacqui's attempt to make an inter vivos gift was ineffective for want of delivery. (B) Jacqui's gift causa mortis was properly made but may now be revoked. (C) Jacqui's attempt to make a gift causa mortis failed due to lack of delivery. (D) Jacqui's attempt to make an inter vivos gift was ineffective due to lack of proper donative intent.

The correct answer is B. This was a gift causa mortis given in contemplation of impending death. This type of gift requires the same elements of donative intent, delivery, and acceptance; all of which were present here. Thus, answer C is incorrect. Courts should accept constructive delivery of the jewelry, accomplished through handing over the key, given that manual delivery would have been difficult. Acceptance of valuable items is generally presumed. So the gift was complete, and had Jacqui actually died, Nala would be the owner of the jewelry. However, with a gift causa mortis, if the donor does not die, the donor may revoke within a reasonable period of time after recovery. Since the gift was not inter vivos, answers A and D are incorrect.

O granted Blackacre "to University so long as tobacco is not consumed in any form on the premises, and if tobacco is ever consumed, Blackacre shall revert to O, her heirs, successors, and assigns." Which one of the following is true? (A) University has a fee simple subject to a condition subsequent. (B) University has a fee simple determinable. (C) University has a contingent remainder. (D) University's interest is void under the Rule Against Perpetuities.

The correct answer is B. University has a present interest, so answer C is incorrect because a remainder is a future interest. Because University's interest could potentially last forever, it is a fee simple. However, because it can end early, it is some form of defeasible fee. There are two possibilities: fee simple subject to a condition subsequent and fee simple determinable. A determinable estate has a duration (e.g., "so long as" or "until") built into the grant, while a conditional estate has a condition attached after an absolute grant ("provided, however" or "on the condition that"). In addition, a determinable fee is typically followed by an automatic reversion on breach, while the conditional fee has an optional termination ("O may retake possession"). The grant in this case is durational and the termination automatic. Therefore, the University has a fee simple determinable, and answer A is incorrect. Answer D is incorrect as the University's interest is not void under the Rule because the Rule applies only to future interests (and by the way, O's interest is also not affected because interests retained by the grantor are not subject to the Rule).

Grandfather received a large amount for the sale of some land, so he decided to share the wealth with his five grandchildren. He had five checks drawn up for $25,000 each and gave them to each grandchild at the family's Thanksgiving celebration. Four of the grandchildren cashed their checks immediately, but Joey decided he needed to get some advice first on what the tax consequences might be before he accepted the gift, so he put the check in a drawer until he could see his accountant. In the meantime, Grandfather learned that Joey said some terrible things about him behind his back at Thanksgiving. He called the bank and asked them to stop payment on the check and then he called Joey and said he was withdrawing the gift. Joey claimed he could not do that because the gift was already complete. Which of the following will be the most likely reason a court would find there was no completed gift? (A) Grandfather lacked donative intent. (B) There was no delivery. (C) Joey did not accept the gift before it was revoked. (D) There was a lack of consideration.

The correct answer is C. A gift requires donative intent, delivery, and acceptance. In this case, Grandfather had donative intent when he gave the checks to the grandchildren. Answer A is therefore incorrect. There was actual delivery of the check and constructive delivery of the cash—he gave each child the means to obtain $25,000 from his bank account. Therefore, answer B is incorrect. Acceptance is usually presumed for gifts of value, but the presumption is rebuttable—in this case we have evidence that Joey was still deciding whether to accept the gift when it was revoked. Answer D is incorrect because consideration is not required in the case of gifts.

Ernst, Frieda, and Gertrude bought Blackacre, a three-bedroom house, and took title as tenants in common. They bought Blackacre as an investment and planned to rent it out. However, Frieda and Gertrude loved the house and decided to move in. Ernst wants Frieda and Gertrude to pay him rent for living in a house that is one-third his, after all. Frieda and Gertrude flat out refuse to do so and tell him he's free to move in with them if he wants to. Which of the following is true? (A) Frieda and Gertrude owe Ernst two-thirds of the rental value of the house. (B) Frieda and Gertrude owe Ernst one-third of the rental value of the house. (C) Frieda and Gertrude do not owe Ernst rent, but the value may be taken into account in determining liability for carrying charges such as insurance in an accounting action. (D) If the three were joint tenants instead of tenants in common, Frieda and Gertrude would have to pay Ernst rent.

The correct answer is C. Any co-tenant may possess the whole property without liability for rent to the other co-tenant. Each co-tenant has the right to possess the property, so the theory is that there is no liability, even though it prevents the property from being used for productive income. The only exception is ouster, when one co-tenant is prevented from exercising his or her right to possession by the other co-tenants. In this case, Frieda and Gertrude explicitly invited Ernst to share possession with them, so there is no case for ouster. Therefore, answers A and B are both incorrect. If ouster had occurred, Ernst would receive one-third of the rental value, not two-thirds. Answer D is incorrect because there is no difference between a joint tenancy and a tenancy-in-common with regard to the rights and responsibilities of the co-tenants. Answer C is correct because even though Ernst can't require the co-tenants in possession to pay rent, the value of their occupation may be taken into account in determining liability for carrying charges. Since they are getting the value from the house, it makes sense that they should pay for the insurance, for example.

Alpha owns Whiteacre, adjacent to Blackacre, owned by Beta. A stream running through Blackacre has begun flooding frequently, and Beta has discovered that the problem is caused by a buildup of brush and mud acting as a dam downstream as the stream runs through Whiteacre. Alpha has refused to take any action and has refused to allow Beta to come on his land to clear the debris. The evidence indicates that correcting the problem would increase the value of both properties. One day, Beta got frustrated and sneaked onto Whiteacre with a chain saw and pickup and cleared the debris. Alpha sued Beta for trespass and sought punitive damages. Which of the following additional facts would least favor Beta in his defense against the trespass suit? (A) Alpha's property was being used for a home improvement store, not a private residence. (B) Flooding would soon imperil Beta's residence, rendering the action a necessity. (C) Beta passed through a gate in the fence around Alpha's property. (D) The stream is large enough to be considered navigable, and Beta's actions were confined to the bed of the stream.

The correct answer is C. As a general rule, trespass is not actionable unless the owner has fenced or posted the property or warned the trespasser to get off of the property; otherwise permission to enter may be implied, especially for open country. Therefore, the fact that Beta passed through a fence would hurt, rather than help, Beta's defense. Answer Awould help Beta's case because the owner's right to exclude is weaker in a public place. Answer B would help Beta also because necessity is recognized as an exception to trespass. Answer D would also help Beta because the bed of a navigable stream is usually owned by the state, not by the private landowner.

Alberto owned Blackacre which was adjacent to Whiteacre, owned by Berta. In 1998, Alberto built his garage without doing a survey and, as a result, part of new garage extended about two feet over the property line onto Whiteacre. In 2005, Alberto died. His will gave Blackacre to his daughter Cara. She moved in and then in 2007 decided to sell Blackacre to Darrell, who promptly moved in. In 2012, Berta was trying to sell Whiteacre to a potential buyer, who discovered that the garage was over the line. In 2012, Berta sued Darrell to require him to move the garage or pay damages. Darrell claims title by adverse possession, which requires ten years of possession in this jurisdiction. Which of the following is true? (A) Darrell cannot meet the statutory period of possession, because he can tack on the possession of Cara but not Alberto. (B) Darrell cannot tack on the possession of either Cara or Alberto to meet the statutory requirement. (C) Darrell should be able to meet the adverse possession requirement by using the possession of Cara and Alberto in addition to his own. (D) Darrell cannot claim adverse possession in any case, because he does not meet the adversity requirement in any jurisdiction.

The correct answer is C. Darrell has been in possession for only five years, so to meet the ten-year statutory period, Darrell will need to tack on the possession of both Alberto and Cara. An adverse possessor can "tack" the possession of previous possessors if they are in privity, which just means that there is some reasonable connection between them that makes it fair to give the current possessor credit for the previous possession. Usually, someone who inherits or is devised property in a will, like Cara, has that connection and so does a good faith purchaser of the property, like Darrell. Thus, answers A and B are incorrect. Tacking would not be allowed only where there is no connection between one possessor and another. So, there does not seem to be any reason a court would deny Darrell the right to tack on those previous periods of possession. D is incorrect because Darrell does not have permission to encroach on Berta's property; therefore, he is adverse. In some jurisdictions, the fact that Darrell did not know he was encroaching might destroy his adversity, but most jurisdictions would deem possession without permission to be adverse.

Alice, the owner of Blackacre, and Bart, the owner of Whiteacre, were neighbors. In 2009, they entered into a reciprocal covenant which stated: "Alice and Bart, their successors and assigns, hereby promise to use the property described below [Blackacre and Whiteacre] for residential purposes only." This agreement was signed by both parties and recorded. In 2015, Alice sold Blackacre to Claire, who intends to build a convenience store on the property. If Bart sues Claire to enforce the covenant, under the common law, which of the following arguments would be Claire's best defense? (A) There was no intent, implied or express, that the covenant run with the land to subsequent owners. (B) There was no vertical privity. (C) There was no horizontal privity. (D) The covenant did not touch and concern the land.

The correct answer is C. Horizontal privity requires there to be a connection between Alice and Bart other than the covenant. At common law, this had to be either a landlord-tenant relationship, a grantor-grantee relationship (if Alice included the covenant when she sold Whiteacre to Bart), or a mutual relationship (if Bart had some other interest in Blackacre, such as an easement, for example). Here, none of those things are true, so horizontal privity, as defined by common law, is lacking here. The other answers do not provide a defense to Bart's claim. Answer A is incorrect because the covenant language indicates an intent to bind successors and assigns in addition to the original parties, which constitutes an express intent to run with the land. At common law, in order for a covenant to run with the land there had to be an intent (either implied or express) that it bind subsequent owners; in addition, there had to be privity (both horizontal and vertical), the covenant had to touch and concern the land, and a subsequent purchaser had to be on notice. Vertical privity requires Claire to succeed to the same interest as Alice, which she did, and therefore answer B is incorrect. Answer D is incorrect because the covenant here is the most common example of one that "touches and concerns" the land because it has to do with how the land is used.

While helping Keisha clean up after a party at her house, Franklin finds a lottery ticket. The ticket turns out to be a winner worth $1 million. The true owner cannot be found, so it looks like Franklin will get the money. Keisha, however, claims the ticket on the basis that it was found on her property. Which of the following would help Keisha's case the least? (A) She hired Franklin to help her clean up. (B) The ticket was found on the counter of her bathroom, so it appears to be mislaid. (C) The ticket was found in the driveway where a lot of cars were coming and going. (D) Keisha had seen the ticket earlier and moved it from the floor to a chair but then was distracted by other tasks.

The correct answer is C. Lost property goes to the first person to possess it, so the question is whether, as the landowner, Keisha had constructive possession of the ticket before Franklin found it. The more private a place is, the more control Keisha has and is more likely to be deemed in constructive possession, whereas in a more public place, the owner has less control and less constructive possession. So, if the ticket was found in the driveway, a fairly public location that Keisha had little control over, it is less likely that she could argue constructive possession. Answer Dindicates more control by Keisha over the item, increasing her constructive possession claim. Answer A will help Keisha, because if she hired Franklin, he would be likely deemed to be her agent and therefore anything he found in the course of his employment would be hers. In Answer B, the ticket is mislaid, rather than lost, which usually favors the owner of the locus (Keisha). C is the only fact that weighs against Keisha's claim.

Sayers enters into a contract to sell Blackacre, an 80-acre parcel in a rural area, to Byers. The contract says that Sayers will provide "marketable title, subject to all easements and encumbrances of record to Blackacre, legally described as W1/2 SE1/4 Section 16 in Guthrie County." Upon examining the title, Byers discovers that Blackacre is encumbered by a gas pipeline easement. In addition, it appears that Sayers does not have record title to about five acres of land (Parcel A) on the edge of the property described in the contract. Sayers shows Byers evidence, however, that he has been in continual, exclusive possession of Parcel A for over 20 years. Given that the adverse possession period in the state is only seven years, Sayers says that he has established good title to Parcel A. Byers refuses to close, alleging a failure of marketable title. Which of the following is true? (A) Both the easement and the Parcel A issue violate the marketable title provisions of this contract. (B) The easement violates the contract's marketable title provision; the Parcel A issue does not. (C) The easement does not violate the contract's marketable title provision; the Parcel A issue does. (D) Neither the easement nor the Parcel A issue violates the contract's marketable title provision.

The correct answer is C. Marketable title requires the seller to establish that she is the record owner, in fee simple absolute, of the property described, without encumbrances. In this case, there are two possible violations of this standard: the easement and the failure to establish record title to Parcel A. Under common law, both of those issues would violate the marketable title standard. However, in this case, Byers chose to accept a contract with slightly different terms, a promise to provide "marketable title subject to all easements and encumbrances of record." Because the pipeline easement is recorded, it does not violate the marketable title provisions of this contract. Thus, answers Aand B are incorrect. The Parcel A issue, however, is not covered by this contract exception. Even though Sayers might, in fact, have title to Parcel A, she does not have "marketable title" because marketable title cannot be subject to a non-frivolous prospect of litigation over ownership. That makes answer D incorrect. Sayers needs to either quiet title to Parcel A before attempting to sell it or get a potential buyer to make it a specific exception to the marketable title provision.

In 1980, Jackson became the owner of Greenacre, a large, undeveloped, wooded tract of about 120 acres in a rural area. Jackson lived about 300 miles away and he visited the property only a few times a year to hunt and repair the fence surrounding the land. In 2000, Pearson lost his home to foreclosure, so he began camping near the middle of Greenacre. He cleared away about a half-acre of brush and trees for his campsite in a ravine surrounded by large bushes. In addition to a large tent, his campsite included an outhouse, a fire pit, a picnic table, and a little storage shed. During a few weeks of the coldest part of each winter, Pearson retreated to a nearby homeless shelter. Jackson did not notice Pearson's site during his periodic visits because he tended to hunt an area near the edge of the tract and often came during Pearson's annual visit to the shelter. In 2017, however, firefighters responding to a small fire near the campsite discovered it and reported it to Jackson. He now wants to evict Pearson. Pearson defends on the basis of adverse possession. The jurisdiction has a ten-year statute of limitations and does not require good faith. Which of the following will be the hardest element for Pearson to meet? (A) Actual possession (B) Continuous possession (C) Open and notorious possession (D) Adversity or hostility

The correct answer is C. Open and notorious possession presents the biggest hurdle. It does not require Jackson to actually KNOW about Pearson's possession, but the fact that it was in a ravine surrounded by bushes would have made it difficult for Jackson to discover. Notorious suggests that the possession is generally known by others, but that is not the case here. Open and notorious suggests conspicuous occupation, whereas this is more hidden or obscure. With regard to A, Pearson's possession is probably sufficient to meet most jurisdictions' requirements for actual possession. He has built a couple of structures and improved the land by clearing it. Unlike the possessors in Van Valkenburgh, whose cultivation of the lot moved about, Pearson stayed in one place for the entire statutory period. Certainly if the jurisdiction had more specific requirements for possession, such as enclosure or cultivation, then Pearson's case would be weakened. Answer B looks possible because Pearson did leave the site every year for several weeks. However, he was still "in possession" because his tent and assorted improvements were still there. Remember the purpose of this requirement is to put the true owner on notice, and the evidence of Pearson's occupation was still there, even when he wasn't. Answer D is incorrect because Pearson did not have permission. If the jurisdiction required good faith, Pearson would have no case, but it doesn't.

In 2005, Dora and Edward inherited Blackacre, their parents' house, as co-tenants when their mother died. Edward had a job in a distant city, but Dora moved in right away. Dora paid the property taxes on the property and took care of any maintenance required. Edward died in 2013, leaving his daughter Fran as his only heir. In 2016, Dora notified Fran that she was selling Blackacre and intended to keep all of the proceeds. "I've been in possession of this house a long time. I'm the one who has taken care of it; your dad didn't do a thing to help. I deserve to have it all to myself." The statute of limitations for ejectment or recovery of possession of property in this jurisdiction is ten years. Dora claims Blackacre by adverse possession. Which of the following is true? (A) Dora should win her case of adverse possession. (B) Dora will lose, because her occupation was not sufficient to constitute "actual possession." (C) Dora will lose, because she did not oust Edward or Fran and therefore her possession was not adverse. (D) Dora will lose, because co-tenants cannot claim adverse possession against each other.

The correct answer is C. Remember that each co-tenant is entitled to possession of the whole premises. It is possible for one co-tenant to adversely possess the premises against the others but only if they clearly indicate their intention to refuse to allow the other co-tenants into possession (i.e. ouster). In this case, Dora possessed the premises, but that was her right as a co-tenant. There is no evidence that she denied Edward's right to possession. Answer A is incorrect because Dora is not an adverse possessor. If she had denied Edward's right to possess, her occupation would have been sufficient to constitute actual possession, so Answer B is wrong. Answer D is wrong because co-tenants can claim adverse possession against other co-tenants but they must establish adversity through ouster.

Sophie entered into a written contract with Developer to buy a condominium, contingent on financing. She particularly wanted Unit 501 because it looked out at the woods and the kitchen colors suited her perfectly. The contract specified that Developer would sell her Unit 501 for a price of $500,000, and it was signed by both parties. Neither party recorded the contract. Just before closing, Developer told Sophie that Unit 501 was no longer available. He sold it to another buyer, Carlos, a few days after entering into the contract with Sophie. Carlos paid $20,000 more, in cash; Developer conveyed to him, and the deed was recorded several days later. Developer believed that Sophie would not be able to qualify for financing, so he was pretty sure he would not have to fulfill his bargain with Sophie and he explained all that to Carlos. Sophie did meet the financing contingency, however, and wants the deal to close. Developer offered her another unit, Unit 505, which is just down the hall. Sophie is not pleased; Unit 505 has a slightly different view of the woods and the colors are different. Plus, she thinks it is not quite as bright and sunny as Unit 501. Developer offers to sell it to her for $450,000, far below the $500,000 asking price, to make up for these differences. Developer argues that because he has already sold Unit 501 to Carlos, he cannot perform the original agreement with Sophie. Which of the following is true in a race-notice jurisdiction? (A) Sophie's remedies are limited to either accepting Unit 505 as a substitute or rescinding the deal entirely. (B) Sophie's remedies are limited to accepting Unit 505, rescinding the contract, or the award of damages for the breach. (C) If she doesn't want to rescind, Sophie should be able to get specific performance of the original agreement, which means that Developer would have to sell her Unit 501. (D) Sophie should be able to get specific performance plus benefit-of-the-bargain damages for Developer's breach.

The correct answer is C. Sophie entered into an enforceable real estate contract with Developer subject to normal contingencies, which she met. Equitable conversion theory indicates Sophie was the equitable owner at the time she entered into the contract. Specific performance is typically available as a remedy for the breach of a real estate contract. The courts reason that, as in the case of Sophie, damages or the substitution of property of the same value will not really remedy the breach due to the unique nature of property. Thus, answers A and B are incorrect. The fact that Developer sold the Unit to Carlos a few days later is irrelevant; at that point, Sophie was the equitable owner of Unit 501. Carlos could possibly get priority under the state's recording act, but since he was told of the contract with Sophie, he can't qualify as a Bona Fide Purchaser. Answer Dis incorrect because specific performance and benefit-of-the-bargain damages are alternative remedies; you can't get both.

Pip found a jewel on the street. He put it in his pocket and decided to take it to show his friend, Miss Havisham. On the way into her house, however, the jewel fell out of Pip's pocket onto the lawn. Some days later, Miss Havisham's niece Estella found it. Pip, Estella, and Miss Havisham are now in litigation over who should get the jewel. Which of the following is true? (A) Estella should get to keep the jewel as the most recent finder. (B) Because the jewel was mislaid by Pip, Miss Havisham should get the jewel as the owner of the locus in quo (i.e., the land where it was found). (C) Pip should get to keep the jewel as the first finder. (D) The jewel should be sold, and all three parties should split the proceeds.

The correct answer is C. The finder of lost property has relative title—good against all the world except the true owner or a previous possessor. Here, Pip was the first finder and thereby acquired this relative title. When Estella found the property, her title was good against all but the true owner or a previous possessor, which was Pip. Therefore, Pip wins against Estella, making A incorrect. Answer B is not correct: The jewel was lost, not mislaid, because Pip did not purposely place it on the lawn. In any case, Miss Havisham's claim would also not be good against Pip, the previous possessor. Answer D might have some equitable appeal, but courts do not usually split lost property between claimants especially where there is clear evidence of first possession.

Assume a jurisdiction follows the majority rule regarding amortization of nonconforming uses. Which of the following ordinances would be least likely to be upheld by the court? (A) An ordinance that rezones the downtown commercial zone to ban adult-oriented businesses, such as adult bookstores and nightclubs, giving them one year to re-locate to other areas on the edge of town (B) An ordinance that rezones a commercial area, currently populated mostly by malls and fast-food restaurants, to eliminate a salvage yard, giving it two years to relocate (C) An ordinance that rezones a residential district to eliminate the only area in town that allows mobile homes, giving mobile home parks one year to cease the use (D) None of these ordinances would be allowed under the majority rule regarding nonconforming uses.

The correct answer is C. The majority rule allows amortization of nonconforming uses as long as the period allowed is reasonable. The reasonableness test includes consideration of the period of time allowed balanced against the amount of investment to be recovered and the degree of conflict the use presents. If the conflict basically constitutes a nuisance, a much shorter period will be allowed because the use could be enjoined by a court. For each of these ordinances, you should consider all three factors. In answer A, there are harms to other businesses and the public and the costs to the adult businesses in relocating are not huge and the owners of the buildings can use them for non-adult commercial uses. In answer B, the conflict between uses is large and the time period is fairly long. In answer C, however, the conflict seems much less—what exactly is the harm caused by a mobile home park? Moreover, the hardship involved is great. These homes can't be relocated elsewhere in town, so they would be worth very little and the burden on the owners would be great. Although you can make an argument for each of these ordinances (making answer Dincorrect), the mobile home ordinance seems least likely to pass the reasonableness test.

Developer owned three lots in a commercial shopping district. She conveyed the middle lot, Lot B, to Pin Me, Inc., which used the lot for a bowling alley. She also conveyed to Pin Me an access easement over Lot A, which was adjacent to the main road. She then conveyed Lot A to a fast food restaurant, QuickEats. She conveyed Lot C to an accountant, Dew Books, who had access through an alley on the other side. Many years later, business was booming for Pin Me. It bought Lot C from Dew Books and planned to expand the bowling alley building by constructing a snack bar on the newly acquired lot. QuickEats sues to enjoin the use of the easement for the expansion, claiming that the proposed construction will result in a misuse of the easement over Lot A. Pin Me thinks QuickEats is trying to prevent a competing food business. Which of the following is true in most jurisdictions? (A) If Pin Me can show that the burden on Lot A will not increase, there is no misuse of the easement. (B) Because users of the easement go to Lot B before continuing to Lot C, there is no misuse of the easement. (C) Even though the expansion is a misuse of the easement, an injunction may not be the appropriate remedy if there is no additional burden on the servient parcel. (D) Because there was no scope limitation in the original easement, there is no misuse by Pin Me.

The correct answer is C. The traditional rule, still the majority, is that an easement cannot be used to benefit a non-dominant parcel. Here, Lot B is the dominant parcel and therefore, using the easement to benefit Lot C is a misuse, pure and simple. Even though the proposed building is physically connected to the existing building on Lot B, it is located on Lot C, so the easement will benefit a non-dominant parcel. So, answers A and B are incorrect because this is a bright-line test; the court will not consider the burden on the servient parcel and the fact that the easement unquestionably can be used to reach Lot B is irrelevant. Similarly, answer D is incorrect because there is no need to explicitly limit the scope of the easement to the dominant parcel. However, as illustrated in Brown v. Voss, 715 P.2d 514 (1986), even though this expansion constitutes a misuse, a court may not order an injunction if there is no real harm to the servient parcel (if the expansion doesn't result in many more customers).

Carl's Car Sales is trying to expand its automobile showroom but cannot get the adjacent landowner, Joan, to sell her land. Joan is using her land for an ice cream shop which she would like to keep. Carl has a friend on the city council, and he complains to her. The city council proposes to condemn the land and transfer it to Carl to provide "economic development" to the area. Joan would like to stop the condemnation. Which one of the following is true under the federal Constitution? (A) The City may not take Joan's property and transfer it to another private party; the condemned land must be owned and used by the public. (B) The City may not condemn Joan's property unless it is in a dilapidated condition. (C) The City may not condemn Joan's property, because its primary purpose is to benefit Carl rather than the public. (D) The City may condemn Joan's property, because the U.S. Supreme Court allows condemnation for economic development.

The correct answer is C. This question concerns the "public use" clause of the Fifth Amendment (applied to state governments and subdivisions through the Fourteenth Amendment). The U.S. Supreme Court has made clear, in cases like Hawai'i Housing Authority v. Midkiff, that "public use" does not require the government to own the property, nor the public to use it. Therefore, answer A is incorrect. Similarly, property may be condemned for many public uses (e.g., highways, schools) even though it is not dilapidated, so answer B is wrong. This case really comes down to the Kelo v. New London test for when condemnation for economic development is constitutional. In that case, the Court held that condemnation for economic development is permissible, but the primary purpose must be to benefit the public rather than another private party. In Kelo, the City was able to meet this test because it had a comprehensive plan for redevelopment and had built up a lot of evidence regarding the economic need for that project. Here, the only motive stated is to benefit Carl and there is no comprehensive plan for redevelopment. Therefore, the best answer is C.

Oscar conveyed Blackacre "to Albert for life, then to Brenda and her heirs." Albert was short of cash, so he sold his interest to a real estate investor named Carl, conveying to him "any and all interest I may have in Blackacre." Carl then died, leaving no will and no heirs. Which one of the following is true? (A) Albert now has his life estate back. (B) Oscar's reversion took effect, giving him the estate. (C) Brenda now has a fee simple absolute. (D) Carl's life estate pur autre vie escheated to the State.

The correct answer is D. Albert has a life estate; Brenda has a vested remainder in fee simple. Albert sold his life estate to Carl, who then had a life estate pur autre vie (measured by Albert's life, that is, not Carl's). When Carl died, his life estate is just like any other property in his estate. It did not terminate because he died—he has an interest measured by Albert's life, not his own. Answer A is incorrect; Albert doesn't get the estate back—he sold his interest to Carl, so he would no more get it back than if he had sold Carl his automobile. Therefore, Carl's life estate pur autre vie would go through his will, if he had one. If not, it would pass by intestate succession to his heirs. If he has no heirs, it would escheat to the state, just like any other property Carl had. There is no reversion to Oscar, making answer A incorrect. Brenda will get her fee simple interest in Blackacre only when Albert dies (or if the life estate owner were to disclaim the interest), thus making answer C incorrect.

Olivia owned a tract of land bordered on the north by Green Road and on the south by Brown Road. In 1996, Olivia built a house on the north half of the property which she called Greenacre. To reach the house, she sometimes used the front driveway leading north out to Green Road, but after heavy snows or heavy rains, Green Road was accessible only with a four-wheel-drive vehicle. Therefore, she often used ''the back way''—a dirt track going over the southern half of the property, which she called Brownacre, leading to Brown Road. In 2006, Olivia sold Greenacre to Andrew and she built and moved into a house on Brownacre. Although the deed said nothing about an easement over Brownacre, Andrew frequently used the dirt track to reach Brown Road, which was a quicker way to reach his workplace. Olivia often saw him and would wave hello. In 2014, a mudslide closed Green Road, and county officials notified Andrew that the damage to the road was severe and might not be repaired for years. Thereafter, Andrew exclusively used the dirt track over Brownacre for ingress and egress. In 2016, Olivia sold Brownacre to Betsy. Soon thereafter, Betsy put up a locked gate across the dirt track and has forbidden Andrew to use it. In this jurisdiction, the statute of limitations for prescription is 10 years. Which of the following is Andrew's best theory for an easement over Brownacre? (A) Estoppel (B) Implied by necessity (C) Prescription (D) Implied by prior use

The correct answer is D. An easement implied by prior use requires a common owner (Olivia); prior use (she used the dirt track to Brown Road herself frequently); reasonable necessity (easier to meet than strict necessity and probably met here due to difficulties in winter), and notice (the dirt track would be visible). Would a reasonable purchaser be justified in assuming that the use of the dirt track was included in the deal? Maybe—this isn't a slam dunk, but it's the best shot for Andrew. The other possibility is an option that wasn't listed: a statutory easement by necessity, which would require Andrew to compensate Betsy for the use of the track and would depend on the statutory requirements in the jurisdiction in which the land is located. Answer A is incorrect because an easement by estoppel requires reasonable reliance on a promise made by Olivia (or Betsy); here there was no such promise. You might think an implied easement by necessity would be possible, answer B, given that Andrew has no real way to get off his property except through Brownacre. However, an implied easement by necessity requires that the necessity arise at the time the property was split into two by the common owner. In 2010, when Olivia sold Greenacre to Andrew, he could use Green Road. The facts indicate that it was sometimes difficult—it required a four-wheel drive vehicle—but courts usually require strict necessity, which these facts do not support. Prescription, answer C, is also no good; the adverse use probably didn't start until 2016. Before then, the use was probably permissive because Olivia saw Andrew using the track and seemed to have no problem with it.

Andy, Barb, and Chet inherited Blackacre, a 120-acre farm, in equal shares as tenants in common. Shortly thereafter, Andy sold his one-third interest to Malls, Inc., which wants the entire parcel so it can build a new shopping mall. Barb does not want the land to be partitioned, particularly not by sale, because she loves the land. Malls, Inc. petitions the court for partition by sale, alleging that dividing the land into three pieces will make it less valuable and that it would be difficult to make the shares come out even. Which of the following is true in most jurisdictions? (A) Barb can set aside the sale of Andy's share to Malls, Inc. because Andy did not have her permission. (B) Malls, Inc. may seek partition only if it can show that remaining in a co-tenancy is impossible or impracticable. (C) A court will order a sale unless Barb can prove that partition in kind will be more beneficial to all parties. (D) Malls, Inc. can get partition by sale if it can show that partition in kind is impracticable or will result in great prejudice to the parties.

The correct answer is D. Answer A is incorrect because any co-tenant can alienate (sell or gift) his or her share of the estate at any time, without permission, unless there is some sort of restraint on alienation on his or her interest. (If there were such as restraint, it would have to be reasonable to be valid, such as that A, B, and C could not sell Blackacre during their lives.) Here there is no restraint, so Andy has the right to alienate without permission. After the sale, Malls, Inc. becomes a tenant in common with Barb and Chet. Answer Bis incorrect because any co-tenant has the right to partition at any time (again, as long as there is no restraint on their interest, which there isn't here). A co-tenant need not show any reason for seeking partition. So, the only real question here is whether the partition will be in kind (physical) or by sale. Typically, courts favor partition in kind because a sale will dispossess the parties of their property. So, the usual test requires a party seeking partition by sale to prove that partition in kind is impracticable or will result in great prejudice to the parties (for example, if it will greatly reduce the value of the land to be broken into smaller pieces). So Answer C is incorrect because Barb does not have the burden of proving partition in kind is more beneficial. Instead, partition in kind will be used unless Malls, Inc. proves it will be impracticable or will result in great prejudice. Therefore, Answer D is the best answer.

Faust wanted to buy Lot 37 of the Berkley Hills Addition from Beowulf. The property contains a two-story house with a detached garage. The address of the property is 1839 Larson Street. Faust took a short tour of the house and loved it immediately. Faust was getting ready to leave for Europe and the two did not have a form contract readily available, so Beowulf simply wrote the terms on a piece of paper as follows: "Faust agrees to purchase 1839 Larson Street from Beowulf for $100,000. Beowulf agrees to provide marketable title. Closing to be two months from today." Both parties signed and dated the document. Upon returning from Europe several weeks later, Faust took a closer look at the house and discovered stained and buckling ceiling plaster in one bedroom, indicating that the roof leaks badly. She now wants to back out of the contract. In the absence of any statutory requirements, which one of the following is true? (A) The initial contract of sale was insufficient to bind Faust to this purchase. (B) The leaky roof constitutes a violation of marketable title because it has a material impact on the property value, allowing Faust to rescind. (C) The leaky roof should have been disclosed to Faust, so she can now rescind. (D) Faust has no valid basis for rescission.

The correct answer is D. Answer A is incorrect because the written memorandum of the agreement meets the Statute of Frauds requirements: Price, Parties, Property. The price and parties are clearly stated; the only question is the description of the property. Although the street address is not a correct legal description, it is sufficient for the contract because it is clear what property is intended. Answer B is incorrect because a physical defect in the property has nothing to do with marketable title. Answer C has to do with the common law doctrine of caveat emptor. At common law, buyers took the property as is and bore the risk of any defects. Modern modifications require disclosure of latent defects, which are problems that cause a material impact on the property's value. The leaky roof is a material defect, but it is not latent. A latent defect is one that is known to the seller but cannot be discovered by reasonable inspection. This defect was patent; it was easy for Faust to see that the plaster was stained and buckling. Therefore, Faust cannot rescind on this basis. Typically, real estate contracts contain an inspection clause allowing the buyer a certain amount of time for a closer inspection to find physical defects. This contract, however, did not include such a clause. In addition, many states have statutes requiring disclosure of certain material defects; the problem asks you to ignore that possibility. Therefore, Faust is stuck with her bargain.

Amy lives on the south side of Route 1 across the road from Jan and Mike Smith, a married couple, who live on the north side. One day she was having coffee with the Smiths on their back deck, and Mike mentioned that he was thinking of selling Blackacre, about 20 acres of land adjacent to Amy's land on the south side of Route 1, which he inherited from his uncle. Amy had always wanted to expand her dairy operation, and told the Smiths that she wanted to buy the land. Amy states that they agreed on a price of $4000/acre, with the exact price to depend on a survey of the land to determine the acreage of the parcel. Amy wrote down on a piece of paper: "Amy Johnson, purchaser, and Jan and Mike Smith, sellers/owners. Property: 20 acres, more or less, on south side of Route 1, Pawnee County. Price: $4000/acre. Survey to be paid for by Amy." Amy and Mike both signed this paper. Jan did not sign. Amy states that Jan said: "That's Mike's department. I've never had anything to do with that land. Whatever he wants is fine." Thereafter, Amy hired a surveyor, and Jan met with the surveyor to show him where the property to be surveyed was located. The survey cost $1000 and showed that the land was about 21 acres. Amy hired an attorney to search the title to Blackacre, which cost her $500. The title search shows the land is held by Mike and Jan Smith, as tenants by the entirety. She also paid $800 for some fence repairs so that the land would be ready to pasture her cattle immediately on closing. At closing, Mike said that the deal would have to be cancelled because Jan refused to sign the deed, complaining that she never wanted to sell and thinks the land will be worth more in the near future. Which of the following is true? (A) Amy can sue Jan for specific performance, because the piece of paper meets the Statute of Frauds. (B) Amy cannot sue Jan for specific performance, because the piece of paper does not contain a correct legal description of Blackacre. (C) Amy cannot sue Jan for specific performance, because the piece of paper does not contain the exact price of the property. (D) Amy should be able to hold Jan to the contract, despite the Statute of Frauds, under a theory of estoppel or part performance.

The correct answer is D. For real estate contracts, the Statute of Frauds requires that there be a memorandum of the agreement signed by the party to be bound, showing the price, the property, and the parties. The piece of paper in this case has an adequate description of the property; it doesn't have to be a legal description but merely sufficient for a court to determine what property was intended. In this case, the property was sufficiently described. Therefore, answer B is incorrect. In addition, although the paper does not indicate the exact price, it is sufficient as long as there is a method by which the price can be determined. Therefore, answer C is incorrect because the method for calculating the price is sufficiently set forth. However, the Statute of Frauds is not met here because Jan did not sign it and she is the "party to be bound." Because the property is held as a tenancy by the entirety, the deal can't go through without her. Therefore, answer A is also incorrect. However, Amy has spent money in reasonable reliance on Jan's agreement to the deal. Her expenditure on the survey, the title work, and the fence repairs should be sufficient to establish either estoppel or part performance. Jan knew Amy was spending the money, yet stood by allowing it to happen.

Which of the following zoning ordinances is most likely to be struck down as a violation of the federal constitution? (A) An ordinance re-zoning a particular tract of vacant land from industrial to commercial, which plaintiffs can show reduces their property value by 30 percent (B) An ordinance giving existing adult-oriented businesses one year to cease operations if they are located within 1,000 feet of a school or church (C) An ordinance requiring a minimum lot size of two acres in an "estate residential" zone on the edge of town, four times the size of any other lot size restriction in town (D) An ordinance restricting certain areas to "single-family" residential use, defined to include any number of persons related by blood, adoption, or marriage but including no more than one set of grandchildren.

The correct answer is D. In Moore v. City of East Cleveland, 431 U.S. 494 (1977), the Court struck down a similar restriction, finding that it impinged on familial relationships without a compelling reason (i.e., Why should two grandchildren who are cousins be banned while two who are sisters are not?) Therefore, the ordinance in answer D is most likely to be struck down. The primary constitutional limits on zoning ordinances include the First Amendment, the Takings Clause, and the Due Process Clause. The ordinance described in answer A is very similar to the one upheld in Euclid v. Amber Realty. As long as the re-zoning has a reasonable relationship to a legitimate city objective, it should be upheld under the Due Process Clause despite the adverse impact on property value. The impact on the property owner is not sufficient to trigger the Takings Clause. Answer Bdescribes a typical amortization ordinance, which are generally upheld if reasonable which this one seems to be given the conflict with other uses. Minimum lot size restrictions, such as those in answer C, are generally upheld despite their exclusionary impacts.

In 2000, Olive owned Blackacre, a modest house on a small lot. In 2005, she properly deeded the property to Andy in exchange for his forgiveness of a loan she couldn't pay back. He agreed to allow her to continue living on Blackacre until she died. In 2006, Andy began serving a prison sentence. In 2007, Olive died. Olive's will specified that all of her property should go to her daughter Bianca. Bianca began living on Blackacre immediately. In 2015, Bianca took a job in a distant city, but she returned to the Blackacre house occasionally to keep it maintained and to visit friends. No one knew about the prior transfer to Andy except Andy himself. When Andy was released from prison in 2017, he brought an action to evict Bianca from Blackacre. In this jurisdiction, the statute of limitations for an action to recover possession of property is seven years. If Bianca tries to establish adverse possession, which of the following requirements will be most likely to undermine her case (A) If the jurisdiction requires continuous possession for the statutory period, her departure to a distant city would not meet the requirement. (B) If the jurisdiction requires open and notorious possession, her occasional return will be insufficient. (C) If the jurisdiction requires good faith adverse possession. (D) If the jurisdiction has a disability statute that allows for tolling, then Olive's challenge may be timely.

The correct answer is D. In some jurisdictions, if the owner is under a disability at the time adverse possession begins, the statute does not begin to run until the disability is removed (or in some jurisdictions, the owner gets some shorter period to act after the disability is removed). If incarceration is defined as a disability under the jurisdiction's statute, Bianca will surely lose her case. In this case, Andy took action to evict immediately after his disability was removed. Answer A is incorrect because Andy was the actual owner of Blackacre in 2005, and, in 2007, when Olive died, Bianca began adversely possessing the property because she did not have Andy's permission and she did not have title. The will was not effective to give her title because any interest Olive may have retained in Blackacre ended at her death. Bianca's possession was continuous for the statutory period of seven years (from 2007-2014). When Bianca took the job in another city, she already had fulfilled the statutory period and, in any event, her absence for long periods may not matter if she maintained possession of the house. Answer Bis also incorrect because Bianca's possession was clearly open and notorious, even though Andy didn't know about it. This requirement means only that someone would see evidence of her possession if he or she looked. It puts the owner on notice; it doesn't require that the owner actually know about the possession. Answer C is incorrect because Bianca did in fact possess in good faith; she truly thought the property was hers pursuant to Olive's will.

Olive conveyed property in trust to be distributed ''to my daughter Amber's first child to graduate from college." At the time of conveyance, Amber is alive and has two children, Cassie and Carter, who are in high school. Which of the following is true? (A) Cassie and Carter have a vested remainder, subject to open, valid under the Rule Against Perpetuities. (B) Cassie and Carter have a contingent remainder, void under the Rule Against Perpetuities. (C) Olive's attempt to create an executory interest is valid under the Rule Against Perpetuities. (D) Olive's attempt to create an executory interest is void under the Rule Against Perpetuities.

The correct answer is D. Olive has attempted to create a springing executory interest in Amber's first child to graduate from college. It is an executory interest because it cuts short the previous estate (in Olive) upon the happening of some event (Amber's child graduating from college). Therefore, answers A and B can't be correct. In fact, Cassie and Carter have nothing until they actually meet the condition. However, Olive's attempt to create an executory interest is void because we will not know who the potential takers are until Amber dies, and then we will not know if they graduate from college within 21 years after that. We can't use Cassie and Carter as validating lives because they may not be the takers; Amber could have another child, years from now, who may be the first one to graduate from college. So, the interest is void under the Rule. Consequently, answer C is incorrect.

Alicia owns Blackacre, adjacent to Whiteacre which is owned by Bao, in an area zoned for agricultural use. Bao decides to use Whiteacre for a large hog lot, with 5,000 hogs, and has received a permit from the state agricultural agency. Alicia had been planning to use Blackacre for a bed and breakfast, which is also allowed in this zone, but she believes the hog operation will result in odor, noise, and flies, making her use infeasible. She wants to sue Bao for an intentional nuisance, and the court finds that Bao's use does constitute a nuisance. This jurisdiction uses the Restatement test to determine whether the remedy should include injunctive relief or damages. The trial elicits the following facts: (1) Alicia's property value will decrease by $75,000 because of the hog lot. (2) Bao has already spent about $50,000 preparing the site for his proposed use including the construction of a confinement building and manure lagoon which cannot be used for any other purpose. (3) Bao's hog lot will employ only a couple of workers. (4) The hog lot will affect not only Alicia's property but also will adversely impact 40 residential lots within a half-mile radius. (5) Bao's lot has been used for many years for crop production and most of the lot can easily be converted back to that use. Which of the facts listed above should the court consider in determining whether to enjoin Bao's use under the Restatement test? (A) Fact #1 only; because the harm to Alicia is substantial, the court should enjoin Bao's proposed use. (B) Facts #1 and #2 only; because the harm to Alicia outweighs the harm to Bao, the court should enjoin Bao's use. (C) All of the facts except #3. (D) All of the facts.

The correct answer is D. The Restatement test requires balancing the equities: the hardship to the plaintiff if the injunction is denied versus the hardship to the defendant if the injunction is granted. In addition, the Restatement indicates that the interests of third parties and the public should be taken into account in determining whether an injunction is appropriate. Restatement (Second) of Torts §§941, 942. Therefore, answer A is incorrect; this answer represents the historic approach to nuisance that the Restatement rejects. The most prominent example is Boomer v. Atlantic Cement Co., 257 N.E.2d 870 (1970) in which the court refused to enjoin a plant despite evidence of significant harm. Answer B is also incorrect because, although these two factors may be important, they do not consider the full extent of the harm. Section 942 says that the harm to the public should also be taken into account, and therefore facts #3 and #4 are relevant as well. In Boomer, for example, the number of people employed by the plant was a key fact. Fact #5 is also relevant because it gives a more complete picture of the hardship to Bao; "hardship" doesn't only mean property value and includes the hardship suffered by both sides of the nuisance equation. Therefore, all of these facts are relevant.

Lance owned an apartment building. Tracy wanted to rent Apartment 2 from Lance, but she noticed there were several problems with it. The sink was clogged, there were some exposed wires where a light fixture was coming loose, and there was a severe mildew odor coming from the closet, where the ceiling seemed to show signs of persistent water damage. Lance acknowledged the problems but said he did not have the time or money to fix the apartment. ''I'll tell you what,'' Lance told Tracy, ''normally this place goes for $600 per month, but given its condition, I can let you have it for $400. But the lease will say that you understand there is no warranty regarding the condition.'' Tracy signed the lease, but several months later, she stopped paying rent, claiming that the mildew odors were overpowering and the other defects made the place unsafe and unhealthy. Which of the following is true? (A) Tracy cannot claim a breach of the implied warranty of habitability, because she did not move out and, therefore, there is no constructive eviction. (B) Tracy cannot claim a breach of the implied warranty of habitability, because she waived it in the lease in exchange for a lower rental price. (C) Tracy can claim a breach of the implied warranty of habitability but only if some of the defects complained of violate the housing code. (D) Tracy has a good case for a breach of the implied warranty of habitability, which can be used in defense if Lance brings an action for unpaid rent.

The correct answer is D. The implied warranty of habitability (IWH) protects residential tenants from conditions that significantly impact safety and health. Answer A is not correct because, unlike the covenant of quiet enjoyment, the IWH does not require the tenant to move out and claim a constructive eviction. Answer B is also incorrect because courts do not allow the IWH to be waived, even where the tenant gets a lower rent in exchange for an attempted waiver. Some Residential Landlord Tenant Acts do allow waivers in limited circumstances, such as single family homes, but not in a case like this. Answer C is too limited; in most jurisdictions, even if violations are not covered in the housing code, as long as the conditions render the premises unsafe or unhealthy, the IWH is breached. In this case, the mildew, exposed wires, and clogged sink seem sufficient to make a good case that the IWH has been violated.

In 2009, Borat borrowed $270,000 from Lender Bank to finance his purchase of Blackacre which appraised for $300,000. Lender took a mortgage on Blackacre to secure the loan, and the mortgage agreement included a "power of sale" provision which allowed Lender to conduct a private foreclosure sale in the event of default. In 2014, Borat defaulted on the loan. The balance of the loan was $255,000 when fees and costs were included. Lender conducted a foreclosure sale after notice to Borat. Lender was the only bidder at the sale and bid $255,000. Lender therefore declared itself to be the owner of Blackacre in exchange for forgiveness of the outstanding balance of the debt. Borat has evidence that the value of Blackacre at the time of the sale was $320,000. Which of the following is most likely true in the majority of jurisdictions? (A) Lender is required to use judicial foreclosure rather than selling the property themselves if Borat objects. (B) The sale can set aside, because Lender was the only bidder. (C) The sale can be set aside, because the sale price was significantly below fair market value. (D) Borat can use the statutory equity of redemption to protect his equity in Blackacre.

The correct answer is D. The question concerns a "power of sale" mortgage which allows the mortgagee to conduct a private foreclosure sale instead of going through the judicial foreclosure process. This is permissible in most jurisdictions, as long as the mortgagee follows the notice provisions and other safeguards to ensure a fair process. Therefore, answer A is incorrect; Lender need not use the judicial foreclosure process as long there is a "power of sale" provision in the mortgage. The safeguards to ensure fairness to Borat do not require that anyone else bid on the property; as long as Lender didn't try to chill bidding and as long as the rules regarding notice were followed, the fact that Lender was the only bidder is not sufficient to set aside the sale, so answer Bis also incorrect. Answer C concerns the sale price. It is true that a sale can be set aside if it "shocks the conscience" of the court; the fact that the price was below fair market value, even "significantly" below, is not sufficient. Answer C is incorrect. A statutory equity of redemption is Borat's best option here; most states allow the mortgagor to redeem the property for a period of time after the sale. If Borat wants to protect his equity in the property, he needs to find funds (perhaps another buyer?) to redeem the property from Lender. Note that there are some jurisdictions that put a higher duty on the mortgagee in a private foreclosure proceeding, requiring that it use best efforts to obtain a fair price. However, that is not the majority rule

Mia owned Blackacre which was bordered to the east by Whiteacre, owned by Nelson. Both properties were bordered by River Road on the south and the Big River on the north. One day, Nelson told Mia he was planning to build a new house further back on Whiteacre, up on a ridge with a good view of the river below. "Unfortunately," he said, "there is no good way to get back there on my property because of a big ravine and wetland. Would you mind if I drive over Blackacre to get back there?" "Sure, I can let you do that, Nelson," Mia replied. The construction company that Nelson hired constructed a dirt road on the extreme eastern edge of Blackacre and used it to get to the building site where it constructed a beautiful house. Nelson moved in and continued to use the dirt road over Blackacre for the next eight years. Nelson then died and his daughter Olivia inherited Whiteacre. She used the road also for another year. Mia then sold Blackacre to Portia. Portia soon got tired of having a stranger drive over her property, so she blocked Olivia's access. Portia claims that Nelson had, at most, a personal, revocable license. Olivia wants to bring an action to establish her right to the easement. The statute of limitations for prescription is seven years in this jurisdiction. Which of the following theories would be most likely to help Olivia in this case? (A) Easement by prescription (B) Easement by estoppel (C) Easement implied by necessity (D) Easement implied by prior use

Mia owned Blackacre which was bordered to the east by Whiteacre, owned by Nelson. Both properties were bordered by River Road on the south and the Big River on the north. One day, Nelson told Mia he was planning to build a new house further back on Whiteacre, up on a ridge with a good view of the river below. "Unfortunately," he said, "there is no good way to get back there on my property because of a big ravine and wetland. Would you mind if I drive over Blackacre to get back there?" "Sure, I can let you do that, Nelson," Mia replied. The construction company that Nelson hired constructed a dirt road on the extreme eastern edge of Blackacre and used it to get to the building site where it constructed a beautiful house. Nelson moved in and continued to use the dirt road over Blackacre for the next eight years. Nelson then died and his daughter Olivia inherited Whiteacre. She used the road also for another year. Mia then sold Blackacre to Portia. Portia soon got tired of having a stranger drive over her property, so she blocked Olivia's access. Portia claims that Nelson had, at most, a personal, revocable license. Olivia wants to bring an action to establish her right to the easement. The statute of limitations for prescription is seven years in this jurisdiction. Which of the following theories would be most likely to help Olivia in this case? A (A) Easement by prescription (B) Easement by estoppel (C) Easement implied by necessity (D) Easement implied by prior use

Alpha owns Whiteacre, adjacent to Blackacre, owned by Beta. A stream running through Blackacre has begun flooding frequently, and Beta has discovered that the problem is caused by a buildup of brush and mud acting as a dam downstream as the stream runs through Whiteacre. Alpha has refused to take any action and has refused to allow Beta to come on his land to clear the debris. The evidence indicates that correcting the problem would increase the value of both properties. One day, Beta got frustrated and sneaked onto Whiteacre with a chain saw and pickup and cleared the debris. Alpha sued Beta for trespass and sought punitive damages. Which of the following rights associated with ownership would most favor Alpha? (A) Right to destroy (B) Right to alienate (C) Right to use (D) Right to exclude

The correct answer is D. The right to exclude others is one of the most fundamental sticks in the bundle of property rights. In cases such as Jacque v. Steenberg Homes and Loretto v. Teleprompter Manhattan CATV, courts have been quite vigilant in protecting this right, even when there is no significant damage to the property invaded. The other rights mentioned are not relevant here. B is incorrect because the right to alienate gives Alpha the right to sell or gift the property. C is incorrect because the right to use the property may have to do with whether Alpha's use interferes with Beta's enjoyment, but it does not stop trespass. And A is incorrect because the right to destroy is typically part of the bundle of property rights but has no application here, as Alpha is not being prevented from destroying his property.


Related study sets

NJVS - Personal Financial Literacy - Unit 6.3: Home and Property Insurance

View Set

中1 理科 生物の観察 身近な生物の生活と種類

View Set

World History Chapter 7 - The Hebrews and Judaism

View Set

Multiple Sclerosis (ATI) & Lupus

View Set

Chapter 3: The Neuronal Membrane at Rest

View Set

Unit 9 Review Electricity Physics

View Set

Week 3 - Fair Credit Reporting Act (FCRA)

View Set

HW 7.1-7.4 Correlation and Causality

View Set

2.1 Section Exercise Set III (Logic)

View Set